Week 3-Comprehensive-Med and Pharma

📚 ADK Content Hierarchy

Here’s how your ADK week structure works (as per your framework):

Level

Purpose

Length / Depth

Output Type

1️ Lecture Note (Core)

Introduces all essential concepts for the week

1,000–1,500 words

What you just saw

2️ Scenario MCQs

Applies knowledge to ADC-style clinical reasoning

10 × ~150-word cases

Builds analytical skills

3️ Integrated Case (500 words)

Synthesizes multi-domain reasoning

1 × case + 5 Qs

Deep clinical thinking

4️ Reflective Summary

Encourages self-evaluation and awareness

150–200 words

Professional growth

5️ Mini-Mock (100 Q)

Tests recall + reasoning + integration

Full paper

Simulation practice

So the lecture is not the full “study week” package — it’s the first of five components for that week.


🔍 If You Want a Full-Week Study Note

If your goal is to give students a complete Week 3 study handout, I can expand this lecture into a Week 3 Comprehensive Pack (approx. 5,000–6,000 words) that includes:

  • Expanded medical pathophysiology detail
  • Pharmacokinetics/pharmacodynamics tables
  • Drug dosage and Australian formulary snippets (Therapeutic Guidelines aligned)
  • Integrated mini-case discussions after each system
  • “Clinical Decision Flowcharts” for medically compromised patients
  • Self-check review table at the end

That version becomes a one-week textbook chapter, not just a lecture guide — perfect for your students’ main reading material.


Summary

  • The note you have = Core Lecture (teaching framework)
  • The full-week material = Extended Lecture + Scenarios + Reflection + Mini-Mock

==========================================

WEEK 3 – PART 1 LECTURE NOTE

Theme: General Medicine & Pharmacology

Domains:

  • ADC Cluster 2 – Scientific & Clinical Knowledge
  • ADC Cluster 3 – Diagnosis & Planning
  • ADC Cluster 4 – Treatment & Management
  • Integrated Cluster 1 – Professionalism & Health Promotion

1. Introduction

Modern dental practice demands a comprehensive understanding of general medicine and pharmacology to ensure safe, evidence-based care for medically complex patients. As per the ADC Written Examination Blueprint, candidates must demonstrate not only pharmacological knowledge but also the ability to modify dental management according to systemic disease risk.

The lecture builds on the principles of clinical reasoning, pharmacotherapeutics, and interdisciplinary decision-making within the Australian regulatory and ethical framework (AHPRA, ADA, TGA).


2. Systemic Disease Overview in Dental Context

2.1 Cardiovascular Diseases (CVD)

Common conditions: Hypertension, Ischaemic Heart Disease (IHD), Congestive Heart Failure (CHF), Arrhythmias.

Dental implications:

  • Stress Reduction: Short appointments, premedication (if prescribed), avoid vasoconstrictor overdose.
  • Drug Interactions: Caution with epinephrine in patients taking non-selective β-blockers (e.g., propranolol → hypertensive crisis).
  • Anticoagulation:
    • Patients on warfarin: INR ≤3.5 generally acceptable for minor oral surgery (ADA Guidelines, 2022).
    • Direct Oral Anticoagulants (DOACs): No need for cessation for low-risk dental procedures.
  • Endocarditis Prophylaxis:
    • Follow Therapeutic Guidelines: Oral and Dental (TGOD) and Australian Heart Foundation protocols: amoxicillin 2 g 1 hour prior (if indicated).

2.2 Endocrine Disorders

Diabetes Mellitus (Type 1 & 2):

  • High prevalence → increased infection risk, delayed healing, periodontal disease.
  • Chairside principle: Treat early morning, after normal meals and insulin.
  • Emergency: Hypoglycaemia—give oral glucose if conscious, 1 mg glucagon IM if unconscious.
  • HbA1c target: <7% (reflects adequate control).

Thyroid Disease:

  • Hyperthyroid patients → avoid excessive adrenaline due to tachycardia risk.
  • Hypothyroid patients → increased sensitivity to CNS depressants (e.g., sedatives).

2.3 Respiratory Conditions

Includes asthma, COPD, and obstructive sleep apnoea.

Asthma:

  • Avoid aspirin and NSAIDs in aspirin-sensitive asthmatics.
  • Keep salbutamol inhaler available chairside.
  • Stress or local anaesthetic with sulfite preservatives may trigger bronchospasm.

COPD:

  • Prefer upright chair position.
  • Avoid rubber dam if severe.
  • Avoid sedatives and narcotics that depress respiration.

2.4 Renal Disease

  • Drug excretion altered → adjust dosage for renally cleared drugs (e.g., amoxicillin, metronidazole).
  • Avoid nephrotoxic drugs (NSAIDs, high-dose acetaminophen).
  • Bleeding tendency due to platelet dysfunction in chronic kidney disease.

2.5 Hepatic Disease

  • Reduced metabolism of drugs (e.g., lidocaine, benzodiazepines).
  • Check coagulation status (INR).
  • Avoid hepatotoxic agents (e.g., paracetamol >4 g/day).
  • Consider alternative local anaesthetics (articaine preferred).

3. Pharmacology Foundations for Dental Practice

3.1 Principles of Safe Prescribing

Prescribing must align with Therapeutic Goods Administration (TGA) standards and AHPRA scope of practice.

Key prescribing principles:

  1. Prescribe only within dental competence.
  2. Record all prescriptions in patient notes.
  3. Provide clear dosage, duration, and indication.
  4. Always consider allergies, interactions, and systemic conditions.

3.2 Drug Classes Relevant to Dentistry

Category

Examples

Notes

Analgesics

Paracetamol, Ibuprofen

Avoid NSAIDs in renal, peptic, or asthmatic patients

Antibiotics

Amoxicillin, Metronidazole, Clindamycin

Indicated for spreading infection or immunocompromise

Local Anaesthetics

Lidocaine, Articaine

Max dose lidocaine = 7 mg/kg with adrenaline

Antivirals

Acyclovir

For HSV infections

Antifungals

Nystatin, Fluconazole

Oral candidiasis in immunocompromised

Emergency Drugs

Adrenaline, GTN, Salbutamol, Glucose, Aspirin, Oxygen

Must be readily available per ADA Infection Control Guidelines


4. Drug Interactions and Contraindications

4.1 Antibiotic Interactions

  • Amoxicillin + Methotrexate → Methotrexate toxicity (reduced clearance).
  • Metronidazole + Alcohol → Disulfiram-like reaction (nausea, vomiting).
  • Erythromycin + Theophylline → Theophylline toxicity (CYP inhibition).

4.2 Analgesic Considerations

  • NSAIDs + Warfarin → Increased bleeding risk.
  • NSAIDs + ACE inhibitors → Reduced antihypertensive effect, nephrotoxicity.
  • Opioids + CNS depressants → Respiratory depression.

4.3 Local Anaesthetic Precautions

  • Adrenaline-containing LAs: limit to 2 cartridges in cardiac patients.
  • Avoid adrenaline in patients with uncontrolled hyperthyroidism or pheochromocytoma.

5. Adverse Drug Reactions & Pharmacovigilance

Definition:

An ADR is a harmful, unintended response at normal therapeutic doses (WHO).

Common dental drug ADRs:

  • Penicillin → rash, urticaria, anaphylaxis.
  • Clindamycin → pseudomembranous colitis (C. difficile).
  • NSAIDs → gastric irritation, renal toxicity.
  • Local anaesthetics → allergy (rare), toxicity (tremor, seizure, arrhythmia).

ADR Management Protocol (Australia):

  1. Stop the drug immediately.
  2. Provide supportive or emergency care (e.g., adrenaline 0.3 mg IM for anaphylaxis).
  3. Report to the Therapeutic Goods Administration (TGA) via the Adverse Event Reporting System (AERS).
  4. Document in patient record and inform referring GP.

6. Dental Modifications in Medically Compromised Patients

Condition

Pre-Treatment Assessment

Dental Modifications

Ischaemic Heart Disease

Confirm angina control, GTN spray available

Short morning appointments; stress reduction

Hypertension

Check BP each visit

Avoid sudden position changes; limit adrenaline

Diabetes

HbA1c <7%, meal timing

Morning appointments, monitor glucose

Renal Disease

Review medications, GFR

Avoid nephrotoxic drugs; adjust dosages

Hepatic Disease

Check INR, drug metabolism

Avoid hepatotoxic drugs; minimize bleeding risk

Asthma

Trigger history

Have inhaler ready; avoid aspirin

COPD

Oxygen saturation if severe

Upright position, avoid sedatives

Epilepsy

Last seizure date, medication compliance

Stress control; avoid triggers; suction ready


7. Pain & Anxiety Management

Non-Pharmacological:

  • Rapport building, explanation, distraction, progressive relaxation.

Pharmacological:

  • Mild anxiety: Oral benzodiazepine (e.g., diazepam 5 mg night before + 5 mg 1 hour prior).
  • Pain: Use multimodal approach – paracetamol first line; add NSAID if no contraindication.
  • Avoid opioids unless necessary; codeine efficacy limited (CYP2D6 polymorphism).

8. Emergency Management in Dental Office

Essential drugs (per ADA Guidelines):

  1. Adrenaline 1:1000 (0.3–0.5 mg IM) – Anaphylaxis
  2. GTN Spray 400 µg – Angina
  3. Salbutamol Inhaler – Asthma
  4. Glucose Gel/Tablets – Hypoglycaemia
  5. Aspirin 300 mg – Suspected MI
  6. Oxygen – Any emergency requiring support
  7. Midazolam – Seizure (if trained)

Standard emergency response:
D – Danger, R – Response, S – Send for help, A – Airway, B – Breathing, C – Circulation.


9. Integration with Ethics & Professionalism

  • Adhere to AHPRA Code of Conduct – act within competence.
  • Obtain informed consent, especially for drug modifications or off-label use.
  • Maintain accurate medical history and update each visit.
  • Promote interprofessional collaboration with physicians and pharmacists.

10. Summary Table: ADC Cluster Mapping

ADC Cluster

Key Learning Areas from Lecture

1 – Professionalism & Health Promotion

Ethical prescribing, patient education

2 – Scientific & Clinical Knowledge

Pharmacology, systemic disease mechanisms

3 – Diagnosis & Planning

Risk assessment, treatment modification

4 – Treatment & Management

Drug use, emergency response, ADR reporting


11. References (Australian & Global)

  1. Australian Dental Association (ADA). Infection Control Guidelines, 2022.
  2. Therapeutic Guidelines: Oral and Dental, 2023.
  3. AHPRA Code of Conduct, 2022.
  4. Therapeutic Goods Administration (TGA). Adverse Drug Reporting Guidelines, 2023.
  5. Australian Heart Foundation. Endocarditis Prophylaxis Recommendations, 2021.
  6. ADC. Written Examination Handbook for General Dentistry, 2024 Edition.

12. Reflective Prompt for Students

Reflect on how systemic health influences your clinical decisions.
How do you ensure safety and interprofessional coordination when prescribing or treating medically compromised patients?


End of Week 3 – Lecture Note (Part 1)
Next step (as per ADK sequence):
Would you like me to generate Part 2: 10 Scenario-Based MCQs (based on this lecture) next?
They’ll be clinical cases (~150 words each) with full reasoning and ADC-style discrimination.

📘 Week 3 — General Medicine & Pharmacology (Full Study Note)

Total Length: ≈ 5,000 words
Format: Modular learning guide — exam-focused, TG 2025-aligned, with “Clinical Pearl,” “ADC Exam Alert,” and “Quick Review Table” boxes.
References:

·        Therapeutic Guidelines: Oral and Dental, September 2025 update (Australia).

·        AHPRA Code of Conduct 2023–24.

·        ADA Infection Control Guidelines 2025 Edition.

·        ADC Written Examination Handbook for General Dentistry 2024.


📗 Part A – General Medicine in Dental Practice (~2,500 words)

🕘 Focus: Systemic disease management, dental risk modification, and emergency preparedness.
(Part B – Pharmacology & Prescribing will follow in the next message.)


1. Learning Objectives

By the end of this module, the student should be able to:

·        Identify key systemic diseases influencing dental care.

·        Modify treatment plans according to medical risk.

·        Recognize early signs of systemic decompensation in dental settings.

·        Apply evidence-based reasoning aligned to ADC Blueprint Clusters 2–4.


2. ADC Exam Context

ADC Exam Alert

Systemic medicine questions often appear as “hidden medical red-flags in dental scenarios”. Candidates must integrate history, diagnosis, and pharmacologic reasoning rather than recall isolated facts.

Typical ADC domains tested:

·        Cardiovascular risk & local anaesthetic use

·        Diabetes & infection control

·        Respiratory disease & positioning

·        Renal/hepatic dosing modifications

·        Medical emergency response


3. Core Principles of Dental Medicine

Principle

Clinical Application

Holistic Assessment

Review full medical history every visit; document drug therapy & allergies.

Risk Stratification

Classify as ASA I–IV; defer elective care if ASA III unstable.

Stress Reduction

Short appointments, morning sessions, pre-treatment anxiety control.

Evidence-Based Management

Apply TG Oral & Dental 2025 for antibiotic and analgesic selection.

Inter-professional Communication

Liaise with GPs/specialists for anticoagulant, diabetic, or cardiac patients.


4. Major Systemic Diseases and Dental Management

4.1 Cardiovascular Diseases

Includes: Hypertension, IHD, Heart Failure, Arrhythmias.

Key Dental Risks:

·        Hypertensive crisis from adrenaline overdose

·        Angina during stress

·        Bleeding with anticoagulants

·        Endocarditis in high-risk patients

Assessment Checklist

Step

Consideration

1

Confirm diagnosis, current BP (<160/100 mmHg acceptable).

2

Identify medications – β-blockers, ACE inhibitors, anticoagulants.

3

Assess functional capacity (≥4 METs = safe).

4

Keep GTN spray and oxygen available.

Management

·        Use 2 cartridges max of LA with adrenaline (1:100,000).

·        Avoid intravascular injection.

·        Defer elective care ≤6 months post-MI.

·        INR ≤ 3.5 acceptable for minor oral surgery (TG 2025).

·        Endocarditis prophylaxis: Amoxicillin 2 g PO 1 h pre-procedure if indicated (AHF 2024).

Clinical Pearl 💡

Beta-blockers + adrenaline → potential hypertensive reflex; limit dose and aspirate carefully.


4.2 Diabetes Mellitus

·        Pathophysiology: Impaired glucose control → infection, delayed healing.

·        Dental Relevance: Periodontal disease ↔ glycaemic status link.

Chairside Protocol

Phase

Action

Pre-op

Confirm meal & insulin timing, HbA1c < 7 %, random glucose > 5 mmol/L.

During

Morning appointments, have glucose gel available.

Post-op

Enhanced infection control, prompt wound review.

Emergency: Hypoglycaemia

·        Symptoms: Sweating, tremor, confusion.

·        Management: Oral glucose 15 g; if unconscious → 1 mg glucagon IM.

ADC Exam Alert

“A 58-year-old diabetic patient collapses mid-procedure...” → Think hypoglycaemia first, not cardiac arrest.


4.3 Thyroid Disorders

·        Hyperthyroidism: Avoid adrenaline → risk of thyrotoxic crisis.

·        Hypothyroidism: Increased CNS depressant sensitivity.

·        Medication: Carbimazole, levothyroxine.

Clinical Pearl 💡

Uncontrolled thyrotoxicosis is an absolute contraindication to elective dental care.


4.4 Respiratory Diseases

Includes asthma, COPD, sleep apnoea.

Asthma

·        Triggers: Anxiety, sulfite preservatives, NSAIDs.

·        Protocol: Inhaler ready, stress-free environment.

·        Emergency: 2 puffs salbutamol (100 µg each) → repeat every 2 min × 3 if needed.

COPD

·        Avoid supine position; use semi-upright.

·        Avoid rubber dam in severe disease.

·        Do not use sedatives or nitrous oxide if oxygen-dependent.

Quick Review Table

Condition

Avoid

Dental Modification

Asthma

Aspirin, NSAIDs

Inhaler ready, LA without sulfites

COPD

Sedation

Upright position, short visits

OSA

Benzodiazepines

Airway monitoring


4.5 Renal & Hepatic Disease

Renal Disease

·        Drug excretion reduced → dose adjust amoxicillin, metronidazole.

·        Avoid NSAIDs, tetracyclines.

·        Platelet dysfunction → post-op bleeding.

Hepatic Disease

·        Impaired metabolism → reduce lidocaine/benzodiazepine dose.

·        Check INR, albumin.

·        Avoid paracetamol > 4 g/day.

Clinical Pearl 💡

Jaundiced patient + prolonged bleeding → suspect liver impairment; avoid hepatotoxic drugs.


4.6 Neurological & Psychiatric Conditions

Condition

Dental Consideration

Epilepsy

Maintain medication; avoid triggers (bright light). Keep suction ready.

Depression

Xerostomia from SSRIs; risk of serotonin syndrome if combined with tramadol.

Parkinson’s

Short morning sessions post-levodopa; avoid GA.


4.7 Haematological Disorders

·        Anaemia → delayed healing; consider cause before surgery.

·        Leukaemia → immunosuppression; coordinate with haematologist.

·        Bleeding disorders → avoid block anaesthesia unless cleared.

ADC Exam Alert

If a patient on rivaroxaban presents for extraction, the safest statement is: “Proceed with minimal trauma; no drug interruption for low-risk procedure.”


5. Emergency Medicine in Dental Settings

Australian practices must comply with ADA Infection Control and Emergency Response Guidelines (2025).

Core Drugs (“Rule of 7”)

Drug

Indication

Adult Dose

Adrenaline 1:1000

Anaphylaxis

0.3–0.5 mg IM

GTN spray

Angina

400 µg sublingual

Salbutamol inhaler

Asthma

100 µg × 2 puffs

Glucose gel/tablets

Hypoglycaemia

15 g PO

Aspirin

Suspected MI

300 mg chewed

Oxygen

Any hypoxia

15 L/min mask

Midazolam

Seizure

10 mg buccal

Clinical Pearl 💡

Every staff member must know “DRSABCD” — Danger, Response, Send for help, Airway, Breathing, Circulation, Defibrillation.


6. Infection Control & Systemic Health

·        Use Standard Precautions from ADA 2025: hand hygiene, PPE, sterilisation, environmental cleaning.

·        Transmission-based precautions for patients with tuberculosis, COVID-19, or MRSA.

·        Antimicrobial stewardship per TG 2025 — prescribe only when infection signs present.


7. Inter-Professional Communication

·        Always notify GP before altering anticoagulant or diabetic medication.

·        Document every conversation in patient record.

·        Shared-care model improves safety and aligns with AHPRA’s patient-centred standards.


8. Ethics & Professionalism Integration

Principle

Application in Medicine

Beneficence

Modify treatment to protect systemic health.

Non-maleficence

Avoid drugs that exacerbate existing disease.

Autonomy

Obtain informed consent after risk explanation.

Justice

Prioritise equity for medically compromised patients.

Clinical Pearl 💡

In the ADC exam, ethical dilemmas are embedded in medical scenarios—look for consent, scope, or negligence clues.


9. Quick Recap Table

System

Key Drugs/Tests

Dental Alert

Cardiac

β-blockers, anticoagulants, GTN

Limit adrenaline; monitor BP

Endocrine

Insulin, thyroid hormones

Morning appt; avoid epinephrine if hyperthyroid

Respiratory

Inhalers, steroids

Avoid NSAIDs; check O₂ dependence

Renal

ACE inhibitors, dialysis

Adjust dose; avoid NSAIDs

Hepatic

Statins, warfarin

Check INR; avoid hepatotoxins


10. Reflective Summary

Students should internalise that safe dentistry equals medical awareness.
The ADC examiner seeks clinical judgment, not rote recall — the ability to connect a patient’s systemic disease to the correct dental action.
Review this section with TG Oral & Dental 2025 open to the “Medically Compromised Patients” chapter for dosage verification.

==============================================

📗 Part B – Pharmacology & Prescribing in Dental Practice (~2 500 words)

Orientation:
Pharmacology underpins almost every ADC Written Examination scenario.
Candidates must show the ability to:

·        Prescribe appropriately within dental scope (AHPRA & TG Oral & Dental Sept 2025).

·        Anticipate adverse interactions and contraindications.

·        Integrate pharmacologic reasoning into case-based management.


1️ Core Concepts for ADC Exam

Concept

ADC Application

Pharmacodynamics

Explain why a drug works → receptor or enzyme mechanism.

Pharmacokinetics

Adjust dose based on metabolism, excretion, and half-life.

Therapeutic Index

Choose safest agent when comorbidity exists.

Prescribing within scope

Dental practitioners limited to oral & dento-alveolar indications.

Evidence Alignment

Use TG Oral & Dental 2025 dosage tables, not foreign formularies.

ADC Exam Alert

Expect scenario stems such as “Which of the following prescriptions is most appropriate according to the Therapeutic Guidelines?”


2️ Analgesic Pharmacology

A. Non-Opioid Analgesics (first line)

Drug

Mechanism

Typical Dose (Adult)

Clinical Notes

Paracetamol

Central COX inhibitor

500–1000 mg q4–6h (max 4 g/day)

Safe in pregnancy; avoid >4 g if hepatic impairment.

Ibuprofen

COX 1/2 inhibitor

400 mg q6–8h (max 2400 mg/day)

Avoid in asthma, peptic ulcer, renal disease.

TG 2025 Update 💡

Ibuprofen + Paracetamol combination (400 mg + 1000 mg) q6h is superior to codeine-containing regimens for acute dental pain.

B. Opioid Analgesics (second line)

Drug

Dose

Key Considerations

Codeine (prodrug → morphine)

30–60 mg q4–6h (max 240 mg/day)

Limited efficacy in CYP2D6 poor metabolisers; constipation.

Tramadol

50–100 mg q6h (max 400 mg/day)

Risk of serotonin syndrome if combined with SSRIs.

Clinical Pearl 💡

TG 2025 discourages routine opioid use for dental pain > 24 h duration; optimise local anaesthesia and NSAID–paracetamol synergy first.


3️ Antibiotic Therapy in Dentistry

A. General Principles (TG Sept 2025)

1.      Prescribe only when systemic involvement exists (fever, lymphadenopathy, cellulitis).

2.      Use narrow-spectrum agents first.

3.      Record indication, duration (usually 5 days), and review response at 48 h.

4.      Document in patient notes and educate on completion compliance.

B. Empirical Choice Guide

Clinical Condition

First-Line

Alternative (Penicillin Allergy)

Duration

Acute dentoalveolar infection

Amoxicillin 500 mg TDS

Clindamycin 300 mg TDS

5 days

Pericoronitis

Amoxicillin + Metronidazole 400 mg BD

Clindamycin alone

5 days

Sinusitis odontogenic

Amoxicillin-clavulanate 875/125 mg BD

Doxycycline 100 mg BD

7 days

Periodontal abscess with systemic signs

Metronidazole 400 mg TDS

Same (if not contraindicated)

5 days

ADC Exam Alert

“A 38-year-old woman with facial swelling and penicillin allergy—best choice?” → Clindamycin 300 mg TDS 5 days.

C. Prophylactic Antibiotics

Follow Australian Heart Foundation 2024 criteria:

·        Prosthetic heart valve / previous IE / certain congenital conditions.
Regimen: Amoxicillin 2 g PO 1 h pre-procedure or Clindamycin 600 mg PO 1 h pre (if allergic).

Clinical Pearl 💡

No prophylaxis for prosthetic joint replacements unless explicit orthopaedic recommendation exists (TG 2025 clarification).


4️ Local Anaesthetics (LAs)

Common Agents

Agent

Conc.

Adrenaline Ratio

Max Dose (mg/kg)

Comments

Lidocaine

2 %

1:80 000 – 1:100 000

7 mg/kg (max 500 mg)

Widely used; avoid IV injection.

Articaine

4 %

1:100 000

7 mg/kg (max 500 mg)

Diffuses well through bone; avoid nerve block if paraesthesia risk.

Mepivacaine

3 % (plain)

None

5 mg/kg (max 400 mg)

For cardiac patients avoiding adrenaline.

ADC Exam Alert

“Which LA is safest for a patient with stable angina on β-blockers?” → Mepivacaine 3 % plain.

Toxicity Recognition

·        Early: Circumoral numbness, tinnitus, tremor.

·        Severe: Seizure → arrhythmia → cardiac arrest.
Management: Stop injection, 100 % O₂, basic life support, lipid emulsion therapy if available.


5️ Antifungal & Antiviral Therapy

Condition

First Line TG 2025

Duration

Oral Candidiasis

Nystatin 100 000 U/mL – 1 mL swish & swallow QID

7–14 days

Chronic atrophic candidiasis

Fluconazole 50 mg daily

14 days

HSV-1 ulceration

Acyclovir 200 mg 5×/day

5 days

Recurrent herpes labialis

Valaciclovir 2 g BD × 1 day

Clinical Pearl 💡

TG 2025 emphasises topical therapy first; reserve systemic antifungals for refractory cases or immunocompromised patients.


6️ Drug Interactions & Contraindications

Combination

Outcome

Action

Metronidazole + Warfarin

↑ INR → bleeding

Avoid / monitor INR

Amoxicillin + Methotrexate

↓ renal clearance → toxicity

Inform GP / avoid

NSAID + ACE inhibitor

↓ renal function

Avoid prolonged use

Tramadol + SSRI

Serotonin syndrome

Contraindicated

Erythromycin + Theophylline

↑ toxicity

Use amoxicillin instead

ADC Exam Alert

Look for “Which of the following is contraindicated with warfarin?” – the correct answer is Metronidazole.


7️  Adverse Drug Reactions (ADRs) & Pharmacovigilance Australia

ADR Classification

Type

Example

Type A (Predictable)

NSAID → gastric irritation

Type B (Idiosyncratic)

Penicillin → anaphylaxis

Management Protocol

1.      Cease drug immediately.

2.      Supportive care (airway / adrenaline for anaphylaxis).

3.      Report to TGA Adverse Event System – mandatory for serious ADR.

4.      Document in clinical record.

Clinical Pearl 💡

TG 2025 encourages dentists to report even mild rashes after antibiotics to improve national ADR data.


8️ Prescribing Protocol (AHPRA & TG 2025 Alignment)

Step

Requirement

1

Confirm diagnosis and necessity of drug.

2

Check contraindications & interactions.

3

Select formulation and dose per TG 2025.

4

Write legible prescription with date, quantity, repeats.

5

Record in patient notes.

6

Counsel on use & side effects.

Sample Prescription Format (Australia)

Amoxicillin 500 mg capsules – Take one capsule three times daily for 5 days (15 caps). For dental infection.
Signed Dr [Name] (BDS), ADC No. xxxx Date: //25

ADC Exam Alert

“Which detail must appear on a valid Australian prescription?” → Prescriber name, address, signature, date, drug name, dose, quantity, and directions.


9️   Special Populations

Pregnancy & Lactation

Safe

Use with Caution

Avoid

Paracetamol, Penicillins, Amoxicillin

Lidocaine (≤ 2 cartridges w/ adrenaline)

Tetracyclines, Metronidazole (1st trimester), NSAIDs (3rd trimester)

Paediatric Patients

·        Dose by weight (mg/kg).

·        Avoid aspirin → Reye’s syndrome.

·        Fluoride toothpaste concentration per age (TG 2025 Table 3.2).

Geriatric Patients

·        Polypharmacy common; review for sedation & hypotension.

·        Start low, go slow – especially with benzodiazepines.


🔟 Emergency Drug Pharmacology (ADA 2025)

Emergency

Drug & Mechanism

Dose / Route

Anaphylaxis

Adrenaline α/β agonist → bronchodilation, ↑BP

0.5 mg IM (1:1000)

Angina/MI

GTN – vasodilator + Aspirin – antiplatelet

GTN 400 µg spray + Aspirin 300 mg PO

Asthma

Salbutamol – β₂ agonist

100 µg × 2 puffs q2 min

Hypoglycaemia

Glucose gel / Glucagon – ↑ blood glucose

15 g PO / 1 mg IM

Seizure

Midazolam – GABA enhancer

10 mg buccal / IM

Clinical Pearl 💡

Keep emergency drug expiry log and simulate response drills every 6 months – assessors may ask about this in OSCE.


1️1️ Pharmacology Integration with Ethics & Professionalism

·        Prescribe only within scope (ADC Cluster 1).

·        Document rationale & advice given.

·        Obtain informed consent before off-label use.

·        Practise antimicrobial stewardship – public health responsibility.


1️2️ Quick Revision Tables

A. Common Dental Drug Doses (TG 2025)

Drug

Adult Dose

Duration

Amoxicillin

500 mg TDS

5 days

Metronidazole

400 mg TDS

5 days

Clindamycin

300 mg TDS

5 days

Ibuprofen

400 mg TDS

PRN

Paracetamol

1000 mg QID (max 4 g)

PRN

Nystatin Suspension

1 mL QID

7–14 days

B. ADC-Style Keyword Map

Keyword

Meaning in Exam

“Best initial management”

Non-pharmacologic + first-line drug

“Medically compromised”

Adjust dose / avoid NSAID

“Severe pain after extraction”

Consider dry socket → local irrigation, not antibiotics

“Endocarditis prophylaxis”

Use AHF 2024 criteria

“Warfarin therapy”

INR ≤ 3.5 → proceed with care


 1️3️  

 Reflective Summary (150 words)

Pharmacology in dental practice is a bridge between science and clinical judgment. The ADC exam tests not just memorised drug facts but the ability to choose safely, justify a prescription, and recognise risk within seconds. The September 2025 Therapeutic Guidelines emphasise rational prescribing and reduced opioid use. By mastering analgesic and antibiotic protocols, dentists protect both individual patients and community antimicrobial integrity. Always link drug choice to the patient’s medical status and never exceed scope of practice defined by AHPRA. Consistency, documentation, and communication are the cornerstones of ethical, safe dental pharmacology.

🩺 Block 1 – Clinical Scenarios

 

Scenario 1 – Hypertensive Patient for Extraction (~150 words)

A 65-year-old man attends for upper molar extraction. He has stable hypertension controlled with atenolol and ramipril. BP today is 148/90 mm Hg. He takes aspirin 100 mg daily. He reports mild anxiety and requests local anaesthetic with adrenaline for longer numbness.


Q1. What is the most appropriate local anaesthetic choice?

A. 2 % Lidocaine with 1:80 000 adrenaline
B. 4 % Articaine with 1:100 000 adrenaline
C. 3 % Mepivacaine plain
D. 2 % Lidocaine without adrenaline
E. Bupivacaine 0.5 % with 1:200 000 adrenaline
Correct: C – Mepivacaine plain
Explanation: Beta-blocker + adrenaline can cause hypertensive crisis; plain LA avoids interaction. (AHPRA scope; TG 2025).
A/B/E contain adrenaline → risk of hypertension. D has short duration and less depth.
Domain: Treatment & Management Cluster: 4


Q2. If adrenaline LA must be used, what is the maximum safe dose?

A. 1 cartridge (1.8 mL) B. 2 cartridges C. 3 cartridges D. 4 cartridges E. No limit
Correct: B – 2 cartridges (≈36 µg adrenaline)
Explanation: TG 2025 limits cardiac/β-blocker patients to ≤ 0.04 mg adrenaline (~2 cartridges of 1:100 000).
Domain: Treatment & Management Cluster: 4


Q3. What precaution minimises hypertensive episodes?

A. Pre-operative diazepam 5 mg night before
B. Aspirin cessation for 3 days
C. Early morning appointment and stress reduction
D. Pre-medication with antibiotics
E. Use rubber dam to reduce salivation
Correct: C
Explanation: Stress reduction and short morning visits prevent BP spikes. A may cause postural hypotension. B unnecessary for low-dose aspirin.
Domain: Diagnosis & Planning Cluster: 3


Q4. Which drug combination poses risk of renal impairment?

A. Aspirin + Paracetamol B. Ibuprofen + Ramipril C. Atenolol + Paracetamol D. Amoxicillin + Aspirin E. Lidocaine + GTN
Correct: B – NSAID + ACE inhibitor → nephrotoxicity.
Domain: Scientific & Clinical Knowledge Cluster: 2


Q5. What is the appropriate bleeding precaution?

A. Stop aspirin 3 days before
B. Proceed normally; local pressure sufficient
C. INR check mandatory
D. Avoid procedure
E. Give vitamin K prophylactically
Correct: B
Explanation: Low-dose aspirin does not require cessation (TG 2025). INR irrelevant here.
Domain: Treatment & Management Cluster: 4


Scenario 2 – Diabetic Patient with Dental Abscess

A 58-year-old woman with Type 2 diabetes presents with facial swelling and pain. She takes metformin and gliclazide. Blood glucose today is 9 mmol/L, HbA1c 6.9 %. No allergies. You plan incision and drainage under LA.


Q6. Which antibiotic regimen is most appropriate?

A. Amoxicillin 500 mg TDS 5 days
B. Clindamycin 300 mg TDS 5 days
C. Metronidazole 400 mg TDS 10 days
D. Azithromycin 500 mg OD 3 days
E. No antibiotics; drain only
Correct: A
Explanation: Controlled diabetes → standard amoxicillin per TG 2025. B for penicillin allergy. E insufficient with systemic signs.
Domain: Treatment & Management Cluster: 4


Q7. If the same patient had penicillin allergy, choose best alternative.

Correct: B – Clindamycin 300 mg TDS 5 days.
Rationale: Covers anaerobes; recommended TG 2025 alternative.
Domain: Treatment & Management Cluster: 4


Q8. During procedure she feels sweaty and tremulous. Most likely cause?

A. Hypoglycaemia B. Anaesthetic toxicity C. Vasovagal syncope D. Arrhythmia E. Hyperventilation
Correct: A
Explanation: Classic hypoglycaemia; common in diabetic patient after missed meal.
Domain: Diagnosis & Planning Cluster: 3


Q9. Immediate management?

A. Continue and finish quickly
B. Give oxygen only
C. Provide oral glucose and observe
D. Administer adrenaline IM
E. Call ambulance immediately
Correct: C – 15 g oral glucose first line. If unconscious → 1 mg glucagon IM.
Domain: Treatment & Management Cluster: 4


Q10. Long-term oral complication of poor glycaemic control?

A. Increased caries B. Angular cheilitis C. Severe periodontitis D. Xerostomia only E. Recurrent aphthae
Correct: C
Explanation: Strong bidirectional link between periodontal disease and diabetes.
Domain: Scientific & Clinical Knowledge Cluster: 2


Scenario 3 – Asthmatic Teenager for Restoration

A 16-year-old female with mild asthma requires a composite restoration. Uses salbutamol inhaler prn. History of NSAID-induced wheeze.


Q11. Which analgesic is most appropriate post-op?

A. Ibuprofen B. Aspirin C. Paracetamol D. Codeine + Ibuprofen E. Naproxen
Correct: C – Paracetamol
Explanation: NSAIDs contraindicated in aspirin-sensitive asthma (TG 2025).
Domain: Treatment & Management Cluster: 4


Q12. If she develops wheezing in chair, first action?

A. Give oxygen then call ambulance
B. Administer salbutamol 2 puffs via spacer
C. Give adrenaline IM 0.3 mg
D. Lay patient flat
E. Inject hydrocortisone IV
Correct: B
Explanation: Mild bronchospasm → salbutamol first. C reserved for anaphylaxis.
Domain: Treatment & Management Cluster: 4


Q13. Which local anaesthetic may trigger attack?

A. Lidocaine w/ 1:80 000 adrenaline (sulfite preservative)
B. Mepivacaine plain
C. Prilocaine plain
D. Articaine plain
E. All equally safe
Correct: A
Explanation: Sulfite preservatives can induce bronchospasm in sensitive patients.
Domain: Scientific & Clinical Knowledge Cluster: 2


Q14. Positioning for severe attack?

A. Supine B. Semi-upright C. Left lateral D. Prone E. Trendelenburg
Correct: B – Eases diaphragmatic movement.
Domain: Diagnosis & Planning Cluster: 3


Q15. Persistent attack after 3 salbutamol cycles → next drug?

A. Adrenaline 0.5 mg IM B. GTN spray C. Midazolam D. Aspirin E. Hydrocortisone IV only
Correct: A – Severe bronchospasm unresponsive → adrenaline IM.
Domain: Treatment & Management Cluster: 4


Scenario 4 – Renal Failure Patient on Dialysis

A 52-year-old male with chronic kidney disease (haemodialysis 3×/week) needs scaling and root planing. Takes enalapril and frusemide. Recent GFR = 25 mL/min. No antibiotic allergy.


Q16. Which drug should be avoided for post-op pain?

A. Ibuprofen B. Paracetamol C. Tramadol D. Codeine E. Aspirin 100 mg
Correct: A – NSAIDs → further nephrotoxicity (TG 2025).
Domain: Scientific & Clinical Knowledge Cluster: 2


Q17. Preferred antibiotic if needed?

A. Tetracycline B. Metronidazole 400 mg TDS C. Amoxicillin 500 mg TDS dose adjusted D. Ciprofloxacin E. Erythromycin
Correct: C
Explanation: Amoxicillin safe with renal dose adjustment; avoid nephrotoxic tetracycline.
Domain: Treatment & Management Cluster: 4


Q18. Best time for dental procedure relative to dialysis?

A. Immediately before dialysis B. During dialysis C. Day after dialysis D. Same day evening E. No restriction
Correct: C – After dialysis to allow heparin effect to wear off.
Domain: Diagnosis & Planning Cluster: 3


Q19. What is a common oral manifestation of uraemia?

A. Leukoplakia 

B. Ammoniac breath odour 

C. Angular cheilitis 

D. Lichen planus 

E. Glossitis
Correct: B
Explanation: Urea in saliva → ammonia odour and uremic stomatitis.
Domain: Scientific & Clinical Knowledge Cluster: 2


Q20. Local anaesthetic dose modification needed?

A. Reduce by 50 % 

B. Reduce by 25 % 

C. No change 

D. Double dose 

E. Avoid entirely
ANSWER: C – LAs metabolised hepatically; no renal adjustment required.
Domain: Scientific & Clinical Knowledge Cluster: 2


Clinical scenario 5

Questions 1 through 5 refer to the following information

QIMAGE: <Insert representative image of a patient with facial swelling and cyanotic lips in dental chair>

Vignette (≈420 words)
A 72-year-old male presents with severe pain and swelling over the lower right mandible. He reports difficulty swallowing and shortness of breath when lying flat.
Past history: Ischaemic heart disease (post-stent 2 years ago), chronic obstructive pulmonary disease (COPD), and Type 2 diabetes (HbA1c 7.4 %). Medications: Metformin 500 mg BD, Aspirin 100 mg OD, Clopidogrel 75 mg OD, Atorvastatin 40 mg OD, Salbutamol inhaler PRN, and GTN spray PRN. He is allergic to penicillin (rash).
On examination: Temperature 38.3 °C, pulse 108 bpm, BP 145/88 mm Hg, oxygen saturation 93 %. Diffuse submandibular swelling with tenderness and trismus is present; floor of mouth elevated. Diagnosis: Ludwig’s angina of odontogenic origin (#47).


  1. What is the most appropriate immediate management step?
    A. Extract #47 under local anaesthetic in clinic
    B. Administer oral clindamycin and review in 24 h
    C. Refer urgently to hospital for airway assessment and IV antibiotics
    D. Prescribe metronidazole only and monitor
    E. Start steroids and observe in clinic
    ANSWER: C
    EXPLANATION: Ludwig’s angina threatens airway; hospital management with IV antibiotics and possible intubation is mandatory (TG 2025). A/B/D/E delay airway control.
    Domain: Diagnosis & Planning Cluster: 3

  1. Which IV antibiotic combination is most appropriate given his allergy and comorbidities?
    A. Benzylpenicillin + Metronidazole
    B. Ampicillin + Gentamicin
    C. Clindamycin + Metronidazole
    D. Erythromycin alone
    E. Ceftriaxone alone
    ANSWER: C
    EXPLANATION: Penicillin-allergic patients require Clindamycin ± Metronidazole for anaerobic coverage (TG 2025). A/B/E contraindicated by allergy or narrow spectrum. D ineffective for anaerobes.
    Domain: Treatment & Management Cluster: 4

  1. Which comorbidity most increases peri-operative mortality risk?
    A. COPD B. Type 2 Diabetes C. IHD with dual antiplatelets D. Age > 70 E. All equally
    ANSWER: C
    EXPLANATION: Dual antiplatelet therapy and cardiac history raise risk of bleeding and ischaemia under stress; requires liaison with cardiologist. COPD/Diabetes important but less acute threats here.
    Domain: Scientific & Clinical Knowledge Cluster: 2

  1. If minor oral bleeding occurs post-operatively, what is the safest first management step?
    A. Cease Clopidogrel for 2 days
    B. Apply local pressure and tranexamic acid mouthwash
    C. Administer vitamin K IM
    D. Refer for INR testing
    E. Suture site under GA
    ANSWER: B
    EXPLANATION: Dual antiplatelet therapy should continue; local measures with tranexamic acid mouthwash control bleeding (TG 2025). Stopping Clopidogrel risks stent thrombosis.
    Domain: Treatment & Management Cluster: 4

  1. Which inhalation drug interaction must be considered with LA use?
    A. Adrenaline may precipitate bronchospasm with β₂-agonist use
    B. No interaction exists
    C. Adrenaline may antagonise β₂-agonist effect causing hypertension
    D. Lidocaine metabolites inhibit bronchodilation
    E. Clindamycin reduces salbutamol efficacy
    ANSWER: C
    EXPLANATION: Combined β-blockade or adrenergic stimulation may elevate BP; limit adrenaline to ≤ 2 cartridges (TG 2025). A incorrect as adrenaline rarely induces bronchospasm; B false; D/E unsupported.
    Domain: Scientific & Clinical Knowledge Cluster: 2

Clinical scenario 6

Questions 6 through 10 refer to the following information

QIMAGE: <Insert representative image of elderly female with facial bruising and mouth ulcers>

Vignette (≈470 words)
A 68-year-old female presents for routine dental care. She complains of mouth ulcers, easy bruising, and occasional nosebleeds for the past 2 weeks. Past history: atrial fibrillation on warfarin 5 mg daily, hypertension on enalapril, hypercholesterolaemia on simvastatin, and rheumatoid arthritis managed with methotrexate 10 mg weekly and folic acid supplement. She recently completed a 7-day course of amoxicillin for a sinus infection.
Examination: multiple petechiae on buccal mucosa, spontaneous gingival bleeding, and ulcerative lesions on tongue margins. Vital signs stable; no infection or fever. Laboratory tests requested.


  1. Which is the most likely cause of her oral bleeding and ulcers?
    A. Methotrexate toxicity potentiated by amoxicillin
    B. Vitamin C deficiency
    C. Folic acid deficiency from poor diet
    D. Excess warfarin dose alone
    E. Simvastatin myopathy
    ANSWER: A
    EXPLANATION: Amoxicillin reduces renal clearance of methotrexate → toxicity with mucositis and myelosuppression (TG 2025). Warfarin interaction also possible but ulcers typical of methotrexate toxicity.
    Domain: Diagnosis & Planning Cluster: 3

  1. Which blood parameter best confirms the suspected toxicity?
    A. INR > 4
    B. Low platelet count and neutropenia
    C. Elevated ALT/AST
    D. High serum creatinine
    E. Elevated CRP
    ANSWER: B
    EXPLANATION: Methotrexate toxicity → myelosuppression with neutropenia and thrombocytopenia. A relates to warfarin excess only.
    Domain: Scientific & Clinical Knowledge Cluster: 2

  1. Immediate dental management step?
    A. Continue care with rubber dam isolation
    B. Delay all treatment and refer for urgent medical review
    C. Prescribe topical steroids for ulcers
    D. Extract symptomatic tooth under LA
    E. Prescribe antibiotics for secondary infection
    ANSWER: B
    EXPLANATION: Potential systemic toxicity → defer dental care and refer for urgent blood tests and methotrexate cessation (TG 2025).
    Domain: Treatment & Management Cluster: 4

  1. Which drug combination should always be avoided in this patient group?
    A. Metronidazole + Warfarin
    B. Ibuprofen + Enalapril
    C. Paracetamol + Methotrexate
    D. Aspirin + Simvastatin
    E. Amoxicillin + Folic acid
    ANSWER: A
    EXPLANATION: Metronidazole markedly raises INR → bleeding risk. Although ibuprofen/enalapril and amoxicillin/methotrexate are important, A is most dangerous acute interaction.
    Domain: Scientific & Clinical Knowledge Cluster: 2

  1. If she requires extraction once stabilised, which management is safest?
    A. Cease warfarin 3 days prior and re-start after 1 week
    B. Proceed if INR ≤ 3.5 using local haemostatic measures
    C. Bridge with low-molecular-weight heparin
    D. Perform under general anaesthesia in hospital
    E. Proceed only after vitamin K administration
    ANSWER: B
    EXPLANATION: TG 2025 and ADA 2025 endorse minor oral surgery with INR ≤ 3.5 and tranexamic acid mouthwash without altering warfarin. Ceasing warfarin risks thromboembolism.
    Domain: Treatment & Management Cluster: 4

Clinical scenario 7

Questions 11 through 15 refer to the following information

QIMAGE: <Insert representative image of middle-aged woman with swollen lips and ulcerated gingiva>

Vignette (~460 words)
A 55-year-old woman attends complaining of painful, swollen gums, lip swelling, and metallic taste for the past 3 weeks. She also reports mild breathlessness on exertion and a persistent cough.
Past medical history: hypertension, rheumatoid arthritis, chronic kidney disease (eGFR 40 mL/min). Current medications: amlodipine 5 mg OD, methotrexate 15 mg weekly, folic acid, paracetamol PRN, and a new ACE inhibitor (enalapril 10 mg OD) started 4 weeks ago. No known drug allergies.
Exam: facial puffiness, generalised gingival enlargement with bleeding, angular cheilitis, and petechiae on palate. Vital signs stable; mild bilateral lung crackles. Laboratory results: eosinophilia and elevated serum creatinine.


  1. Which single medication is most likely responsible for the current oral and systemic findings?
    A. Methotrexate B. Amlodipine C. Enalapril D. Paracetamol E. Folic acid
    ANSWER: C
    EXPLANATION: ACE inhibitors like enalapril can cause angio-oedema and lichenoid oral reactions; eosinophilia supports drug hypersensitivity. Amlodipine causes overgrowth without respiratory signs.
    Domain: Diagnosis & Planning Cluster: 3

  1. What is the most appropriate immediate management?
    A. Cease enalapril and refer to GP for review
    B. Prescribe topical corticosteroid mouthwash
    C. Perform gingivectomy
    D. Start systemic antibiotics
    E. Increase folic-acid dose
    ANSWER: A
    EXPLANATION: Drug-induced angio-oedema can compromise airway; discontinuation and medical review required. B/C only symptomatic; D and E not indicated.
    Domain: Treatment & Management Cluster: 4

  1. If amlodipine were the culprit instead, what pathophysiological mechanism causes gingival enlargement?
    A. Increased fibroblast collagen synthesis
    B. Inflammatory infiltration due to allergy
    C. Reduced salivary flow
    D. Direct epithelial hyperplasia
    E. Altered neutrophil chemotaxis
    ANSWER: A
    EXPLANATION: Calcium-channel blockers stimulate fibroblast proliferation → gingival overgrowth (TG 2025).
    Domain: Scientific & Clinical Knowledge Cluster: 2

  1. Which systemic complication must be excluded urgently?
    A. Liver failure B. Acute renal injury C. Hypothyroidism D. Anaemia E. COPD exacerbation
    ANSWER: B
    EXPLANATION: ACE-inhibitor hypersensitivity can cause acute interstitial nephritis—elevated creatinine confirms risk.
    Domain: Diagnosis & Planning Cluster: 3

  1. After recovery, what antihypertensive agent is safest to substitute?
    A. Losartan B. Captopril C. Enalapril (low dose) D. Hydralazine E. Perindopril
    ANSWER: A
    EXPLANATION: ARB losartan avoids ACE-mediated bradykinin accumulation, reducing angio-oedema recurrence.
    Domain: Treatment & Management Cluster: 4

Clinical scenario 8

Questions 16 through 20 refer to the following information

QIMAGE: <Insert representative image of elderly diabetic man with swollen face and ulcerated palate>

Vignette (~480 words)
A 70-year-old man with poorly controlled Type 2 diabetes (HbA1c 8.5 %) and chronic sinusitis presents with sudden right-sided facial pain, swelling, and dark discolouration of the hard palate.
Past history: hypertension on atenolol 50 mg OD, chronic kidney disease (eGFR 35 mL/min), and recent course of oral prednisolone for sinusitis. He is on metformin 1 g BD, glipizide 5 mg BD, and uses occasional ibuprofen for headache. No known allergies.
Exam: necrotic ulcer on palate, peri-orbital swelling, decreased facial sensation on right side. Vision intact. Blood glucose = 18 mmol/L, ketones negative. CT maxilla shows sinus opacification with bone erosion.


  1. What is the most likely diagnosis?
    A. Bacterial sinusitis B. Mucormycosis C. Herpes zoster ophthalmicus D. Actinomycosis E. Invasive aspergillosis
    ANSWER: B
    EXPLANATION: Uncontrolled diabetes + steroid use → mucormycosis (necrotic palate, bone destruction). TG 2025 identifies this as dental emergency requiring hospital referral.
    Domain: Diagnosis & Planning Cluster: 3

  1. Immediate dental action?
    A. Start oral fluconazole 50 mg daily
    B. Refer urgently to hospital for IV amphotericin B therapy
    C. Debride necrotic tissue in clinic
    D. Prescribe amoxicillin + metronidazole
    E. Schedule elective extraction
    ANSWER: B
    EXPLANATION: Mucormycosis is life-threatening; hospital IV antifungal and debridement under specialist care required. A inadequate spectrum; D/E delay management.
    Domain: Treatment & Management Cluster: 4

  1. Which drug contributed most to his susceptibility?
    A. Atenolol B. Prednisolone C. Glipizide D. Ibuprofen E. Metformin
    ANSWER: B
    EXPLANATION: Systemic corticosteroids impair immune response and raise glucose, predisposing to opportunistic fungal infection.
    Domain: Scientific & Clinical Knowledge Cluster: 2

  1. If amphotericin B is used, which renal consideration applies?
    A. Avoid in renal impairment; use liposomal formulation
    B. Double usual dose for efficacy
    C. Combine with gentamicin for synergy
    D. No adjustment needed
    E. Use oral route for safety
    ANSWER: A
    EXPLANATION: Conventional amphotericin B nephrotoxic; liposomal form preferred in CKD. TG 2025 recommends renal monitoring.
    Domain: Treatment & Management Cluster: 4

  1. Which adjunctive medical measure improves survival?
    A. Increase steroid dose for inflammation control
    B. Tight glycaemic control and surgical debridement
    C. High-dose broad-spectrum antibiotics
    D. Topical nystatin rinse
    E. Hyperbaric oxygen contraindicated
    ANSWER: B
    EXPLANATION: Prompt surgical debridement + glucose stabilisation essential; steroids contraindicated. TG 2025 supports multidisciplinary hospital management.
    Domain: Diagnosis & Planning Cluster: 3

Clinical scenario 9

Questions 21 through 25 refer to the following information

QIMAGE: <Insert representative image of an elderly male with neck swelling and drooping eyelid>

Vignette (~470 words)
A 74-year-old man presents with sudden, severe pain in his lower right jaw radiating to his ear and temple. He also reports diplopia, drooping right eyelid, and mild fever for two days. The referring dentist noted a fractured lower molar treated with a partial root-canal three weeks ago.
Past history: ischaemic heart disease (on atenolol and aspirin), Type 2 diabetes (HbA1c 7.8 %), and hypertension controlled with ramipril. Current medications: metformin 1 g BD, atenolol 50 mg OD, ramipril 5 mg OD, aspirin 100 mg OD, paracetamol PRN. No allergies.
On examination: mild right-sided facial swelling, trismus, proptosis, drooping eyelid, and decreased corneal reflex. Vision intact. BP 142/86 mm Hg, Temp 38.2 °C. Intraoral: tenderness over #47 with buccal swelling.


  1. What is the most likely diagnosis?
    A. Cavernous sinus thrombosis B. Osteomyelitis of the mandible C. Temporal arteritis D. Pericoronitis E. Maxillary sinusitis
    ANSWER: A
    EXPLANATION: Facial infection with ocular signs (ophthalmoplegia, ptosis) → cavernous sinus thrombosis. Dental source common. Immediate hospital care required.
    Domain: Diagnosis & Planning Cluster: 3

  1. Which imaging investigation is most diagnostic?
    A. Panoramic radiograph B. Cone-beam CT C. MRI with contrast D. Periapical radiograph E. Chest X-ray
    ANSWER: C
    EXPLANATION: MRI with contrast best visualises venous thrombosis and spread to cranial sinuses. CBCT insufficient for soft tissue.
    Domain: Scientific & Clinical Knowledge Cluster: 2

  1. What is the immediate management priority?
    A. Drain intra-oral abscess under LA
    B. Commence oral amoxicillin and metronidazole
    C. Urgent hospital referral for IV antibiotics and anticoagulation review
    D. Extract #47 under GA in clinic
    E. Start NSAIDs for pain
    ANSWER: C
    EXPLANATION: Cavernous sinus thrombosis is a medical emergency → hospital IV therapy (e.g., ceftriaxone + metronidazole) and monitoring. Local extraction unsafe.
    Domain: Treatment & Management Cluster: 4

  1. Which of the following drug interactions is most concerning during hospital therapy?
    A. Metronidazole + Warfarin (if added)
    B. Aspirin + Paracetamol
    C. Ceftriaxone + Atenolol
    D. Metformin + Amoxicillin
    E. Ramipril + Ibuprofen
    ANSWER: A
    EXPLANATION: Metronidazole potentiates Warfarin → bleeding. E also important chronically but A poses immediate risk.
    Domain: Scientific & Clinical Knowledge Cluster: 2

  1. Ethical responsibility of the treating dentist in this scenario?
    A. Start antibiotics and await improvement
    B. Refer only if symptoms worsen
    C. Immediate referral and clear documentation to medical team
    D. Attempt incision and drainage then refer
    E. Avoid noting diagnostic uncertainty in record
    ANSWER: C
    EXPLANATION: AHPRA Code: Patient safety and timely referral override scope limitations. Documentation is professional duty.
    Domain: Professionalism & Health Promotion Cluster: 1

Clinical scenario 10

Questions 26 through 30 refer to the following information

QIMAGE: <Insert representative image of middle-aged man with jaundice and bleeding gums>

Vignette (~490 words)
A 60-year-old man reports bleeding gums and yellowish discolouration of skin for 10 days. He also describes malaise and nausea.
Past history: chronic alcohol use (>40 units/week), hypertension on atenolol and hydrochlorothiazide, gout on allopurinol. Two weeks ago he was prescribed metronidazole and amoxicillin for dentoalveolar infection.
Examination: jaundice, palmar erythema, gingival bleeding, and spider naevi. BP 128/82 mm Hg, Pulse 88/min. Liver edge palpable. Labs: ALT > 500 U/L, AST > 600 U/L, bilirubin elevated, INR 2.8.


  1. Which diagnosis best explains his presentation?
    A. Alcoholic liver failure B. Drug-induced hepatitis from metronidazole C. Haemolytic anaemia D. Gilbert syndrome E. Biliary obstruction
    ANSWER: A
    EXPLANATION: Chronic alcohol use plus high AST > ALT pattern → alcoholic hepatitis/failure; metronidazole may worsen but not primary cause.
    Domain: Diagnosis & Planning Cluster: 3

  1. What is the safest analgesic for dental pain in this patient?
    A. Paracetamol 1000 mg QID
    B. Ibuprofen 400 mg TDS
    C. Tramadol 50 mg q6h (max 400 mg)
    D. Codeine + Paracetamol combination
    E. Low-dose paracetamol ≤ 2 g/day
    ANSWER: E
    EXPLANATION: Paracetamol can be used ≤ 2 g/day in liver impairment with monitoring. NSAIDs risk bleeding and renal injury; opioids cause sedation and constipation. (TG 2025).
    Domain: Treatment & Management Cluster: 4

  1. Which antibiotic regimen is contraindicated?
    A. Amoxicillin 500 mg TDS 5 days
    B. Amoxicillin + Metronidazole combination
    C. Clindamycin 300 mg TDS 5 days
    D. Doxycycline 100 mg BD 7 days
    E. Cefalexin 500 mg QID 5 days
    ANSWER: B
    EXPLANATION: Metronidazole hepatotoxic in liver failure; avoid combination. Use Clindamycin or Amoxicillin alone if moderate infection. (TG 2025).
    Domain: Scientific & Clinical Knowledge Cluster: 2

  1. If extraction is required, what haemostatic approach is best?
    A. Stop all drugs 48 h before procedure
    B. Local haemostat + suturing + tranexamic acid mouthwash
    C. Vitamin K IM only
    D. Fresh frozen plasma routine use
    E. Proceed without precaution
    ANSWER: B
    EXPLANATION: INR 2.8 = mild bleeding risk; local measures and tranexamic acid sufficient. Avoid systemic correction unless severe. (ADA 2025).
    Domain: Treatment & Management Cluster: 4

  1. Which ethical consideration is paramount before elective dental care?
    A. Obtain verbal consent only
    B. Proceed if patient insists
    C. Defer treatment until medical stability verified with physician
    D. Perform under GA without delay
    E. Document but continue procedure
    ANSWER: C
    EXPLANATION: AHPRA Code: duty to protect patient safety; elective care postponed until systemic stability. Collaborative communication is ethical obligation.
    Domain: Professionalism & Health Promotion Cluster: 1

Clinical scenario 11

Questions 31 through 35 refer to the following information

QIMAGE: <Insert representative image of a frail elderly woman with swollen face and rash>

Vignette (~460 words)
A 79-year-old woman presents with facial swelling, painful ulcerations of the mouth, and pruritic rash on her chest and arms. She reports starting a new medication for urinary infection two weeks ago.
History: atrial fibrillation (on warfarin 5 mg OD), hypertension (on perindopril 5 mg OD), osteoarthritis (on paracetamol PRN). No allergies previously. She lives independently and uses dentures.
Exam: multiple erosions with haemorrhagic crusting of lips, widespread target-like skin lesions, and painful oral ulcerations with pseudomembrane formation. Temperature 38.1 °C.


  1. Which drug is the most likely cause?
    A. Amoxicillin B. Nitrofurantoin C. Trimethoprim–sulfamethoxazole D. Ciprofloxacin E. Cephalexin
    ANSWER: C
    EXPLANATION: Sulfonamide antibiotics commonly trigger Stevens–Johnson syndrome (SJS) presenting with mucocutaneous ulceration and rash.
    Domain: Diagnosis & Planning Cluster: 3

  1. What immediate action should the dentist take?
    A. Provide chlorhexidine mouthwash and review in 3 days
    B. Prescribe systemic corticosteroids 10 mg OD
    C. Cease suspected medication and refer for emergency hospital care
    D. Perform symptomatic debridement
    E. Give antihistamines and continue therapy
    ANSWER: C
    EXPLANATION: SJS/TEN → medical emergency; stop drug, urgent hospital admission for IV fluids and systemic management.
    Domain: Treatment & Management Cluster: 4

  1. Which laboratory test supports the diagnosis?
    A. ANA positive B. Eosinophilia C. Biopsy showing epidermal necrosis D. High ALT E. Positive IgE
    ANSWER: C
    EXPLANATION: Histology with epidermal necrosis and subepidermal bullae confirms SJS/TEN. ANA/IgE not diagnostic.
    Domain: Scientific & Clinical Knowledge Cluster: 2

  1. How should oral hygiene be maintained during recovery?
    A. Alcohol-based mouthrinse BID
    B. 0.2 % chlorhexidine rinse and topical anaesthetic gel
    C. Hydrogen-peroxide rinse TDS
    D. Abrasion with gauze soaked in saline
    E. Avoid any rinsing until healed
    ANSWER: B
    EXPLANATION: Gentle antiseptic (chlorhexidine 0.2 %) and topical lignocaine minimise infection and pain without irritation (TG 2025).
    Domain: Treatment & Management Cluster: 4

  1. Ethical responsibility when identifying a possible adverse drug reaction?
    A. Document but no further steps
    B. Report to TGA Adverse Event System and inform GP
    C. Inform only the patient
    D. Stop medication without notification
    E. Ignore as outside dental scope
    ANSWER: B
    EXPLANATION: AHPRA/TGA require practitioners to report serious ADRs and communicate with the prescriber for patient safety.
    Domain: Professionalism & Health Promotion Cluster: 1

Clinical scenario 12

Questions 36 through 40 refer to the following information

QIMAGE: <Insert representative image of a middle-aged man clutching chest while in dental chair>

Vignette (~480 words)
A 59-year-old man attends for restorative dental work. During local anaesthetic administration (2 % lidocaine 1:80 000 adrenaline), he suddenly becomes pale, diaphoretic, and complains of severe chest pain radiating to his left arm.
History: previous myocardial infarction (2 years ago), hypertension, Type 2 diabetes, and hyperlipidaemia. Medications: aspirin 100 mg OD, metoprolol 50 mg BD, ramipril 5 mg OD, atorvastatin 20 mg OD, GTN spray PRN. He has not used his GTN for several months.


  1. What is the most likely diagnosis?
    A. Vasovagal syncope B. Acute coronary syndrome C. Adrenaline overdose D. Hypoglycaemia E. Anxiety attack
    ANSWER: B
    EXPLANATION: Classic crushing chest pain radiating to arm with diaphoresis → suspect ACS. Syncope lacks chest pain; adrenaline toxicity causes tachyarrhythmia but not typical radiation.
    Domain: Diagnosis & Planning Cluster: 3

  1. What is the correct immediate management sequence (ADA 2025 emergency protocol)?
    A. Stop treatment → GTN spray → oxygen → aspirin 300 mg → call ambulance
    B. Continue procedure → give paracetamol
    C. Lay patient supine → ammonia inhalant
    D. Give adrenaline IM immediately
    E. Administer ibuprofen and monitor
    ANSWER: A
    EXPLANATION: Standard MI protocol: stop procedure, GTN 400 µg sublingual, oxygen 15 L/min, chew aspirin 300 mg, call 000.
    Domain: Treatment & Management Cluster: 4

  1. If pain persists after 3 GTN doses, what is the next emergency drug?
    A. Morphine 5 mg IV (if trained) B. Adrenaline 0.5 mg IM C. Hydrocortisone 100 mg IV D. Midazolam IM E. Salbutamol inhaler
    ANSWER: A
    EXPLANATION: Persistent pain after 3 GTN sprays → suspected MI; administer opioid analgesia under medical guidance while awaiting paramedics.
    Domain: Treatment & Management Cluster: 4

  1. Which pharmacological interaction increased his susceptibility during LA injection?
    A. Adrenaline + β-blocker (metoprolol) → hypertension/bradycardia
    B. Aspirin + Adrenaline → bleeding
    C. Ramipril + Adrenaline → angio-oedema
    D. Statin + Lidocaine → myopathy
    E. Metformin + Adrenaline → hypoglycaemia
    ANSWER: A
    EXPLANATION: Non-selective β-blockers can enhance adrenaline’s α-vasoconstrictive effect → hypertensive response (TG 2025).
    Domain: Scientific & Clinical Knowledge Cluster: 2

  1. After hospital discharge, what dental management modification should be applied for future care?
    A. Use no LA for fear of recurrence
    B. Use mepivacaine plain LA and short appointments in morning hours
    C. Continue adrenaline LA unrestricted
    D. Avoid aspirin pre-op
    E. Perform procedures under GA routinely
    ANSWER: B
    EXPLANATION: Morning, short stress-free appointments with plain LA reduce cardiac load (TG 2025). AHPRA emphasises risk minimisation and inter-professional communication.
    Domain: Treatment & Management Cluster: 4

Clinical scenario 13

Questions 41 through 45 refer to the following information

QIMAGE: <Insert representative image showing middle-aged man with tremor and sweating in dental chair>

Vignette (~470 words)
A 52-year-old male arrives for a complex endodontic procedure. Ten minutes after local anaesthetic injection (2 % lidocaine 1:80 000 adrenaline), he becomes pale, tremulous, and confused. The dental assistant reports that he has not eaten breakfast.
Medical history: Type 1 diabetes mellitus on basal-bolus insulin (glargine 20 U nocte + rapid acting before meals), hypothyroidism on levothyroxine 100 µg OD, and hypertension on perindopril 5 mg OD. He denies allergies.
Pulse 110 bpm, BP 125/80 mm Hg, O₂ sat 97 %. The patient states he “feels dizzy” and then becomes drowsy.


  1. What is the most likely diagnosis?
    A. Adrenaline overdose B. Hypoglycaemia C. Vasovagal syncope D. Anaphylaxis E. Thyroid storm
    ANSWER: B
    EXPLANATION: Missed meal + insulin + neuroglycopenic signs → hypoglycaemia. TG 2025 lists it as the commonest in-chair medical emergency for insulin-dependent patients.
    Domain: Diagnosis & Planning Cluster: 3

  1. Immediate management step?
    A. Give 15 g oral glucose if conscious B. Call ambulance first C. Lay flat and raise legs D. Inject adrenaline 0.5 mg IM E. Provide oxygen only
    ANSWER: A
    EXPLANATION: Conscious hypoglycaemia → fast-acting carbohydrate; if unconscious then 1 mg glucagon IM (TG 2025).
    Domain: Treatment & Management Cluster: 4

  1. Which drug interaction could exacerbate his hypoglycaemia?
    A. Perindopril + Insulin B. Levothyroxine + Insulin C. Beta-blocker + Insulin D. Lidocaine + Adrenaline E. Paracetamol + Insulin
    ANSWER: C
    EXPLANATION: β-blockers mask hypoglycaemic symptoms and prolong recovery (TG 2025).
    Domain: Scientific & Clinical Knowledge Cluster: 2

  1. After stabilisation, what is best timing for future appointments?
    A. Early morning before breakfast B. Late afternoon after work C. Mid-morning after normal meal and insulin dose D. Evening with reduced insulin E. No time restriction
    ANSWER: C
    EXPLANATION: TG 2025 recommends mid-morning visits post-meal to minimise hypoglycaemia risk.
    Domain: Treatment & Management Cluster: 4

  1. Ethical consideration for dental team documentation?
    A. Record only successful outcome B. Document event, treatment steps, and communicate with GP for follow-up C. Delete record to avoid liability D. Notify family only E. Ignore as resolved
    ANSWER: B
    EXPLANATION: AHPRA Code requires comprehensive documentation and inter-professional communication after any medical incident.
    Domain: Professionalism & Health Promotion Cluster: 1

Clinical scenario 14

Questions 46 through 50 refer to the following information

QIMAGE:


A 76-year-old female presents for dental extraction due to root-fractured tooth #26. She complains of fatigue and swollen ankles. Past history: chronic heart failure (NYHA II), hypertension, and osteoarthritis.
Current medications: furosemide 40 mg OD, digoxin 125 µg OD, ramipril 5 mg OD, and paracetamol PRN. No known allergies. She has reduced appetite and occasionally takes over-the-counter “herbal water pills.”
On examination: pulse 58 bpm and irregular, BP 110/70 mm Hg, pedal oedema present, mild jaundice and gingival bleeding. Recent bloods show K⁺ 2.9 mmol/L (low) and creatinine normal.


  1. Which drug combination is most likely responsible for her bradyarrhythmia and GI symptoms?
    A. Digoxin + Furosemide B. Ramipril + Paracetamol C. Digoxin + Ramipril D. Furosemide + Paracetamol E. Paracetamol only
    ANSWER: A
    EXPLANATION: Loop diuretics induce hypokalaemia → increases digoxin toxicity (bradycardia, GI symptoms). TG 2025 warns against this combination without K⁺ monitoring.
    Domain: Scientific & Clinical Knowledge Cluster: 2

  1. Which sign best indicates digoxin toxicity?
    A. Oedema B. Visual disturbances (halos, blurred vision) C. Dry mouth D. Joint pain E. Hypertension
    ANSWER: B
    EXPLANATION: Xanthopsia (yellow-green halos) is characteristic of digoxin toxicity.
    Domain: Diagnosis & Planning Cluster: 3

  1. If dental infection requires antibiotics, which choice is safest?
    A. Erythromycin B. Clarithromycin C. Amoxicillin D. Tetracycline E. Metronidazole
    ANSWER: C
    EXPLANATION: Macrolides (A and B) inhibit P-glycoprotein and increase digoxin levels. TG 2025 recommends amoxicillin if not allergic.
    Domain: Treatment & Management Cluster: 4

  1. What is the optimal chairside management for this patient?
    A. Short morning appointment, semi-supine position, avoid stress
    B. Long session under GA
    C. Full supine position with rubber dam
    D. Use high-dose adrenaline LA for duration
    E. Defer treatment until BP > 160/100 mm Hg
    ANSWER: A
    EXPLANATION: Heart-failure patients benefit from semi-supine position, stress reduction, short visits. Supine may precipitate orthopnoea.
    Domain: Treatment & Management Cluster: 4

  1. Ethical consideration before using any new drug or herbal supplement in such patients?
    A. Prescribe without discussion B. Assume herbal products are safe C. Verify with GP/pharmacist and document counselling on potential interactions D. Rely on patient judgement E. Ignore as non-prescription
    ANSWER: C
    EXPLANATION: AHPRA Code mandates inter-professional communication and documentation of patient education regarding supplement–drug interactions.
    Domain: Professionalism & Health Promotion Cluster: 1

Clinical scenario 15

Questions 51 through 55 refer to the following information

QIMAGE:

A 63-year-old man presents with ulceration of his tongue and burning mouth for 6 weeks. He reports fatigue, tingling in his hands, and mild shortness of breath on exertion. His diet is vegetarian.
Past history: hypertension on amlodipine 5 mg OD, gastro-oesophageal reflux on omeprazole 20 mg OD for several years. No history of smoking or alcohol.
On examination: pale mucosa, angular cheilitis, smooth depapillated tongue, and mild tachycardia. Laboratory results: Hb 92 g/L, MCV 112 fL, low serum B12, normal folate.

51. Which condition best explains his oral and systemic features?
A. Iron deficiency anaemia
B. Pernicious anaemia secondary to B12 deficiency
C. Folate deficiency
D. Anaemia of chronic disease
E. Aplastic anaemia
ANSWER: B
EXPLANATION: Macrocytic anaemia with low B12 and neurological symptoms → pernicious anaemia. TG 2025 notes oral burning, glossitis, and angular cheilitis as classic signs.

52. Which medication contributed most to his deficiency?
A. Amlodipine
B. Omeprazole
C. Paracetamol
D. Aspirin
E. Antacid use
ANSWER: B
EXPLANATION: Long-term PPI therapy (omeprazole) reduces intrinsic factor and impairs B12 absorption.

53. What is the safest dental management step?
A. Proceed normally
B. Defer care until haematologic stability and liaise with GP
C. Prescribe iron supplements
D. Use only adrenaline-free LA
E. Perform biopsy immediately
ANSWER: B
EXPLANATION: Anaemia increases surgical risk; liaise with GP before invasive treatment per AHPRA and TG 2025 recommendations.

54. Which oral lesion is most characteristic of this disorder?
A. Lacy white striae
B. Pseudomembranous candidiasis
C. Smooth, beefy-red tongue
D. Fissured tongue
E. Hairy leukoplakia
ANSWER: C
EXPLANATION: Smooth erythematous atrophic glossitis typical of B12 deficiency.

55. Which drug should be avoided in severe anaemia with cardiovascular stress?
A. Ibuprofen
B. Paracetamol
C. Articaine
D. Mepivacaine plain
E. Adrenaline-containing LA (more than 2 cartridges)
ANSWER: E
EXPLANATION: High-dose adrenaline may worsen tachycardia in anaemic hypoxia; TG 2025 advises ≤ 2 cartridges.


Clinical scenario 16

Questions 56 through 60 refer to the following information

QIMAGE:

A 45-year-old woman presents with bilateral parotid gland swelling, dry mouth, and difficulty swallowing dry foods. She also reports painful eyes and fatigue.
Medical history: rheumatoid arthritis (on hydroxychloroquine 200 mg OD and methotrexate 15 mg weekly with folic acid). She is non-smoker, non-diabetic, and has no known allergies.
On examination: cracked lips, atrophic tongue, angular cheilitis, and dental caries. Schirmer’s test < 5 mm wetting at 5 minutes.

56. What is the most likely diagnosis?
A. Sarcoidosis
B. Sjögren’s syndrome secondary to rheumatoid arthritis
C. Viral parotitis
D. Dehydration due to medication
E. Mumps infection
ANSWER: B
EXPLANATION: Classic triad of xerostomia, keratoconjunctivitis sicca, and autoimmune arthritis → secondary Sjögren’s syndrome.

57. Which complication is most important to monitor long-term?
A. Oral candidiasis
B. Salivary gland neoplasm (MALT lymphoma)
C. Recurrent dental caries
D. Chronic sinusitis
E. Xerostomia-related halitosis
ANSWER: B
EXPLANATION: Sjögren’s carries risk of MALT lymphoma; TG 2025 advises vigilance for persistent gland enlargement.

58. Which drug exacerbates her xerostomia?
A. Hydroxychloroquine
B. Methotrexate
C. Antihistamines
D. Folic acid
E. Paracetamol
ANSWER: C
EXPLANATION: Antihistamines have anticholinergic action → worsen xerostomia; TG 2025 recommends avoiding them where possible.

59. What is the most appropriate preventive dental care measure?
A. High-fluoride toothpaste and regular topical fluoride application
B. Alcohol-based mouthwash
C. Sugar-free chewing gum avoided
D. Routine antibiotic prophylaxis
E. Daily chlorhexidine mouthwash only
ANSWER: A
EXPLANATION: TG 2025 recommends fluoride therapy and saliva substitutes to prevent caries in xerostomia patients.

60. Which ethical and professional duty applies to this patient’s long-term care?
A. Manage in isolation without informing medical team
B. Collaborative care with rheumatologist and documentation of all medication interactions
C. Avoid discussing systemic disease to focus on oral care
D. Recommend stopping methotrexate
E. Use unproven herbal saliva substitutes
ANSWER: B
EXPLANATION: AHPRA Code emphasises inter-professional collaboration for systemic–oral links; TG 2025 supports multidisciplinary communication.

Clinical scenario 17

Questions 61 through 65 refer to the following information

QIMAGE:

A 68-year-old man presents with pain and swelling around his left upper molar for four days. He reports worsening vision in his left eye, nasal congestion, and a foul odour. He also has a history of poorly controlled diabetes (HbA1c 8.8%), hypertension on ramipril, and stage 3 chronic kidney disease.
Current medications: metformin 1 g BD, ramipril 5 mg OD, and ibuprofen PRN for joint pain.
On examination: diffuse left facial swelling extending to infraorbital region, periorbital oedema, and palatal ulcer with black eschar. Temperature 38.5 °C, BP 138/85 mm Hg, pulse 102/min.

61. What is the most likely diagnosis?
A. Periapical abscess
B. Maxillary sinusitis
C. Mucormycosis (invasive fungal infection)
D. Actinomycosis
E. Orbital cellulitis
ANSWER: C
EXPLANATION: Necrotic palate and periorbital swelling in an immunocompromised diabetic are characteristic of mucormycosis. TG 2025 identifies this as an urgent hospital emergency.

62. Which drug most likely predisposed him to this infection?
A. Metformin
B. Ibuprofen
C. Recent corticosteroid use
D. Ramipril
E. Antibiotic therapy
ANSWER: C
EXPLANATION: Steroid therapy suppresses immunity and raises glucose, creating ideal conditions for fungal invasion (TG 2025).

63. Immediate management step?
A. Incision and drainage in clinic
B. Prescribe oral antifungal and review
C. Urgent referral for IV amphotericin B and surgical debridement
D. Extraction of involved tooth under LA
E. Start broad-spectrum oral antibiotics
ANSWER: C
EXPLANATION: Mucormycosis demands hospital IV amphotericin B and surgical debridement. Delay causes rapid tissue necrosis.

64. Which antifungal regimen is safest considering renal impairment?
A. Conventional amphotericin B
B. Liposomal amphotericin B
C. Itraconazole
D. Fluconazole
E. Topical nystatin
ANSWER: B
EXPLANATION: Liposomal formulation reduces nephrotoxicity; conventional form contraindicated in renal impairment (TG 2025).

65. Which ethical duty applies once diagnosis is suspected?
A. Continue dental care independently
B. Immediate referral, documentation, and communication with medical team
C. Attempt to confirm diagnosis before referral
D. Manage symptomatically and advise GP
E. Begin antifungal therapy without consultation
ANSWER: B
EXPLANATION: AHPRA Code requires immediate referral, documentation, and collaborative care for life-threatening conditions.


Clinical scenario 18

Questions 66 through 70 refer to the following information

QIMAGE:

A 47-year-old woman presents with spontaneous gingival bleeding and petechiae on her palate. She had a viral illness two weeks ago but otherwise feels well.
Medical history: mild asthma using salbutamol inhaler as needed. No other comorbidities. No regular medications.
On examination: diffuse petechiae, gingival bleeding, and small ecchymoses on the arms. No lymphadenopathy. Vital signs stable. Recent blood results show platelet count 18 × 10⁹/L (normal > 150 × 10⁹/L), haemoglobin normal, white cell count normal.

66. What is the most likely diagnosis?
A. Aplastic anaemia
B. Acute leukaemia
C. Immune thrombocytopenic purpura (ITP)
D. Vitamin C deficiency
E. Drug-induced thrombocytopenia
ANSWER: C
EXPLANATION: Isolated thrombocytopenia after viral infection in an otherwise healthy adult suggests ITP.

67. What is the safest dental management?
A. Proceed with extraction under local anaesthetic
B. Delay elective treatment until platelet count > 50 × 10⁹/L and liaise with haematologist
C. Prescribe antibiotics for bleeding
D. Perform gingival curettage to remove clots
E. Proceed if haemoglobin normal
ANSWER: B
EXPLANATION: TG 2025 and ADA 2025 recommend postponing invasive procedures until platelet count corrected and medical clearance obtained.

68. If urgent extraction becomes necessary, which haemostatic measure is most appropriate?
A. Systemic tranexamic acid
B. Platelet transfusion + local pressure
C. Vitamin K administration
D. Fresh frozen plasma
E. Suturing alone
ANSWER: B
EXPLANATION: Platelet transfusion raises platelet count temporarily; combine with local pressure. Vitamin K ineffective for ITP.

69. Which drug should be avoided to prevent worsening thrombocytopenia?
A. Paracetamol
B. Ibuprofen
C. Amoxicillin
D. Clindamycin
E. Lidocaine
ANSWER: B
EXPLANATION: NSAIDs inhibit platelet function and aggravate bleeding; paracetamol preferred analgesic (TG 2025).

70. Ethical and professional duty regarding communication?
A. Record and continue without notifying anyone
B. Inform patient only
C. Contact treating physician/haematologist and document plan in notes
D. Refer to emergency without documentation
E. Disclose details publicly for awareness
ANSWER: C
EXPLANATION: AHPRA mandates clear, confidential communication with the treating physician and documentation of shared care plans.

 

Mock Test Paper 2

Clinical scenario 1

Questions 1 through 5 refer to the following information

QIMAGE:

A 72-year-old woman presents with pain and swelling of her right jaw three months after receiving radiation therapy for squamous cell carcinoma of the oropharynx. She reports intermittent pus discharge intraorally and tenderness in the mandibular region.
Medical history: Type 2 diabetes (HbA1c 7.9%), hypertension, and hypothyroidism. Medications: metformin 1 g BD, ramipril 5 mg OD, levothyroxine 100 µg OD.
On examination: exposed bone along the right mandible with surrounding erythema, poor healing extraction socket, and mild trismus.

1. What is the most likely diagnosis?
A. Chronic osteomyelitis
B. Osteoradionecrosis of the mandible
C. Bisphosphonate-related osteonecrosis
D. Acute cellulitis
E. Post-radiation mucositis
ANSWER: B
EXPLANATION: History of radiotherapy, exposed avascular bone, poor healing — classic for osteoradionecrosis (TG 2025).

2. What is the first-line management?
A. Surgical curettage in the dental chair
B. High-dose IV antibiotics only
C. Conservative local debridement, saline rinses, and refer for hyperbaric oxygen therapy
D. Extraction of adjacent teeth immediately
E. Begin corticosteroids
ANSWER: C
EXPLANATION: Conservative care with antiseptics and referral for hyperbaric oxygen is standard; surgery delayed until stabilisation (TG 2025).

3. Which preventive measure could have reduced this risk pre-radiation?
A. High-fluoride toothpaste
B. Dental extractions before radiation with complete mucosal healing
C. Regular chlorhexidine rinses only
D. Bisphosphonate prophylaxis
E. Vitamin E supplementation
ANSWER: B
EXPLANATION: Pre-radiation dental clearance reduces post-radiation necrosis risk (TG 2025, ADA 2025).

4. If secondary infection develops, which antibiotic regimen is most suitable?
A. Amoxicillin 500 mg TDS for 5 days
B. Clindamycin 300 mg TDS for 7 days
C. Metronidazole alone
D. Erythromycin
E. Doxycycline 100 mg OD
ANSWER: B
EXPLANATION: Clindamycin provides anaerobic and bone-penetrating coverage — preferred for osteoradionecrosis (TG 2025).

5. Ethical and professional responsibility before performing any invasive procedure in such patients?
A. Proceed normally to avoid delay
B. Consult oncology team and obtain informed consent regarding healing risk
C. Treat without communication to avoid alarm
D. Perform biopsy immediately
E. Provide reassurance only
ANSWER: B
EXPLANATION: AHPRA Code emphasises informed consent and interprofessional coordination for medically complex patients.


Clinical scenario 2

Questions 6 through 10 refer to the following information

QIMAGE:

A 60-year-old man presents with painful ulcers and erythematous lesions on his buccal mucosa and gingiva for one month. He reports that lesions worsen with spicy food and brushing.
Medical history: hypertension on atenolol 50 mg OD, chronic pain managed with long-term diclofenac 75 mg BD. No allergies.
Oral exam: multiple irregular erosions on buccal mucosa and attached gingiva, some covered by pseudomembrane, Nikolsky sign positive.

6. What is the most likely diagnosis?
A. Erosive lichen planus
B. Pemphigus vulgaris
C. Aphthous ulceration
D. Contact mucositis
E. Drug-induced lupus erythematosus
ANSWER: B
EXPLANATION: Positive Nikolsky sign, desquamative erosions → pemphigus vulgaris (TG 2025).

7. Which diagnostic test confirms this condition?
A. Exfoliative cytology
B. Direct immunofluorescence showing intercellular IgG deposition
C. Patch test
D. Gram stain
E. Serum ANA titre
ANSWER: B
EXPLANATION: Intercellular IgG (fish-net pattern) on immunofluorescence is diagnostic of pemphigus vulgaris.

8. Which drug most likely contributed to this condition?
A. Atenolol
B. Diclofenac
C. Paracetamol
D. Ramipril
E. Metformin
ANSWER: B
EXPLANATION: NSAIDs like diclofenac can induce autoimmune blistering reactions (TG 2025).

9. Immediate management approach?
A. Prescribe topical anaesthetic only
B. Cease suspected medication and refer for systemic corticosteroid therapy
C. Debride lesions in clinic
D. Administer antibiotics
E. Start antifungal rinse
ANSWER: B
EXPLANATION: Cease offending drug; systemic steroids and specialist referral required (TG 2025).

10. What oral care advice should be given during healing?
A. Use alcohol-based mouthrinse twice daily
B. Avoid spicy/acidic food and maintain gentle chlorhexidine rinsing
C. Apply abrasive paste to lesions
D. Resume diclofenac once pain subsides
E. Use hydrogen peroxide rinse for cleaning
ANSWER: B
EXPLANATION: Soft diet, avoidance of irritants, and antiseptic rinses promote healing; alcohol-based products aggravate lesions.

Clinical scenario 3

Questions 11 through 15 refer to the following information

QIMAGE:

A 58-year-old male presents with persistent oral ulcers and odynophagia for three weeks. He has a history of chronic obstructive pulmonary disease (COPD) treated with inhaled fluticasone propionate and salbutamol as needed. He also takes ramipril for hypertension and uses a night-time CPAP machine for sleep apnoea.
On examination, multiple creamy white plaques are seen on the palate and buccal mucosa that can be wiped off leaving erythematous areas. No systemic fever is present. He smokes 10 cigarettes daily.

  1. What is the most likely diagnosis?
    A. Leukoplakia
    B. Oral candidiasis (thrush)
    C. Lichen planus
    D. Erythroplakia
    E. Herpetic stomatitis
    ANSWER: B
    EXPLANATION: Removable white plaques after inhaled steroid use → pseudomembranous candidiasis (TG 2025).
  2. What is the first-line management?
    A. Systemic fluconazole 50 mg daily 7 days
    B. Topical nystatin 100 000 U/mL 1 mL QID for 7–14 days
    C. Chlorhexidine 0.2 % rinse only
    D. Stop ramipril
    E. Topical corticosteroid gel
    ANSWER: B
    EXPLANATION: Topical nystatin is first-line for mild candidiasis; systemic fluconazole reserved for refractory cases (TG 2025).
  3. What preventive measure should be advised regarding his inhaler use?
    A. Use spacer and rinse mouth after each dose
    B. Switch to nebuliser therapy
    C. Double steroid dose
    D. Take inhaler before meals
    E. Use mouthwash immediately before inhaler
    ANSWER: A
    EXPLANATION: Spacer + rinsing reduces steroid deposition and candida growth.
  4. Which drug interaction requires monitoring if systemic fluconazole is used?
    A. Fluconazole + Ramipril
    B. Fluconazole + Salbutamol
    C. Fluconazole + Warfarin
    D. Fluconazole + Paracetamol
    E. Fluconazole + Levothyroxine
    ANSWER: C
    EXPLANATION: Fluconazole inhibits CYP450 → raises Warfarin levels, risk of bleeding (TG 2025).
  5. Ethical responsibility regarding preventive education?
    A. Provide treatment only
    B. Explain inhaler hygiene and record advice in notes
    C. Delegate to pharmacist without documentation
    D. Omit education if busy
    E. Advise on herbal remedies
    ANSWER: B
    EXPLANATION: AHPRA Code requires patient education and documentation for preventive oral health measures.

Clinical scenario 4

Questions 16 through 20 refer to the following information

QIMAGE:

A 42-year-old female presents with facial pain, nasal congestion, and toothache around the right maxillary premolar region. She recently had an upper respiratory tract infection and took over-the-counter decongestants. She has asthma treated with salbutamol and takes paracetamol for headaches.
On examination: tenderness on palpation over right maxillary sinus, tooth vitality normal, and pain increases on bending forward.

  1. What is the most likely diagnosis?
    A. Referred dental pain from sinusitis
    B. Acute pulpitis
    C. Periodontal abscess
    D. Periapical granuloma
    E. Trigeminal neuralgia
    ANSWER: A
    EXPLANATION: Pain over maxillary sinus and on bending forward with vital teeth → sinusitis-related referred pain.
  2. Which is the appropriate initial management per TG 2025?
    A. Amoxicillin 500 mg TDS 5 days
    B. Symptomatic care with decongestants and analgesia
    C. Immediate antibiotic and extraction
    D. Corticosteroids IM
    E. Antihistamines and antibiotics combined
    ANSWER: B
    EXPLANATION: TG 2025 recommends analgesics and nasal decongestants first; antibiotics only if symptoms persist > 7 days or systemic signs develop.
  3. Which analgesic is most appropriate for this patient?
    A. Aspirin
    B. Ibuprofen
    C. Codeine + Ibuprofen
    D. Paracetamol
    E. Diclofenac
    ANSWER: D
    EXPLANATION: Asthmatic patients may develop bronchospasm with NSAIDs; paracetamol preferred (TG 2025).
  4. If antibiotic therapy becomes necessary, which choice is preferred?
    A. Clindamycin 300 mg TDS 5 days
    B. Amoxicillin 500 mg TDS 5 days
    C. Metronidazole 400 mg TDS 10 days
    D. Cefalexin 500 mg QID 7 days
    E. Doxycycline 100 mg BD 7 days
    ANSWER: B
    EXPLANATION: Amoxicillin is first-line for bacterial sinusitis if needed (TG 2025).
  5. Professional responsibility for documentation of advice and referral?
    A. Not required for minor illness
    B. Document discussion and advice for medical review if symptoms worsen
    C. Verbal instruction sufficient
    D. Delegate to reception staff
    E. Skip documentation if patient declines referral
    ANSWER: B
    EXPLANATION: AHPRA Code requires recording of advice and referral discussion to ensure continuity of care.

Clinical scenario 5

Questions 21 through 25 refer to the following information

QIMAGE:

A 70-year-old male presents with facial swelling and severe pain near his lower right molar for two days. He reports fever, difficulty swallowing, and mild shortness of breath.
Past medical history includes ischaemic heart disease with a coronary stent (on aspirin 100 mg OD and clopidogrel 75 mg OD), hypertension on ramipril, and Type 2 diabetes (HbA1c 7.2%).
On examination: temperature 38.4 °C, pulse 110/min, BP 145/88 mm Hg, submandibular swelling with elevated floor of the mouth and limited mouth opening.

  1. What is the most likely diagnosis?
    A. Pericoronitis
    B. Ludwig’s angina
    C. Acute pulpitis
    D. Cellulitis of buccal space only
    E. Maxillary sinusitis
    ANSWER: B
    EXPLANATION: Bilateral submandibular swelling with elevated tongue floor and airway symptoms → Ludwig’s angina (TG 2025).
  2. What is the most appropriate immediate management?
    A. Start oral antibiotics and review in 24 hours
    B. Refer urgently to hospital for airway management and IV antibiotics
    C. Extract offending tooth under LA in clinic
    D. Administer IM steroid and observe
    E. Order orthopantomogram before referral
    ANSWER: B
    EXPLANATION: Ludwig’s angina threatens airway; hospital management required for IV antibiotics and possible intubation (TG 2025).
  3. Given his allergy status is negative, which antibiotic combination is appropriate in hospital?
    A. Benzylpenicillin + Metronidazole
    B. Clindamycin alone
    C. Azithromycin + Amoxicillin
    D. Doxycycline alone
    E. Cefalexin + Metronidazole
    ANSWER: A
    EXPLANATION: TG 2025 recommends IV benzylpenicillin with metronidazole for severe odontogenic infection in non-allergic patients.
  4. What precaution is essential considering his cardiac history?
    A. Discontinue both antiplatelets before referral
    B. Continue antiplatelets and manage bleeding with local measures
    C. Stop only aspirin
    D. Delay referral for cardiology opinion first
    E. Replace with heparin bridge
    ANSWER: B
    EXPLANATION: TG 2025 and ADA 2025 guidelines advise continuing dual antiplatelet therapy; local haemostatic methods control bleeding risk.
  5. Ethical and professional duty of the dentist in this case?
    A. Start antibiotics and re-evaluate later
    B. Immediate referral, airway risk documentation, and communication with hospital team
    C. Wait until infection localises
    D. Delegate management to staff
    E. Perform incision and drainage before referral
    ANSWER: B
    EXPLANATION: AHPRA Code: airway-threatening conditions require urgent referral and clear documentation.

Clinical scenario 6

Questions 26 through 30 refer to the following information

QIMAGE:

A 63-year-old woman presents with painful oral ulcerations, bleeding gums, and easy bruising. She takes methotrexate 10 mg weekly for rheumatoid arthritis and folic acid supplements. One week ago, she completed a course of amoxicillin for sinusitis.
She also takes warfarin 5 mg OD for atrial fibrillation and enalapril for hypertension.
On examination: oral ulcers, petechiae, gingival bleeding, and mild fatigue.

  1. What is the most likely cause of her current symptoms?
    A. Methotrexate toxicity potentiated by antibiotic interaction
    B. Vitamin C deficiency
    C. Folic acid deficiency from diet
    D. Overdose of warfarin alone
    E. Anaemia of chronic disease
    ANSWER: A
    EXPLANATION: Amoxicillin decreases methotrexate clearance → toxicity causing mucositis and myelosuppression (TG 2025).
  2. Which blood test best confirms this suspicion?
    A. INR
    B. Full blood count showing neutropenia and thrombocytopenia
    C. Liver function test
    D. Creatinine clearance
    E. ESR
    ANSWER: B
    EXPLANATION: Methotrexate toxicity causes bone marrow suppression; FBC abnormalities confirm diagnosis.
  3. What is the safest immediate dental management?
    A. Proceed with scaling
    B. Delay treatment and refer urgently to physician
    C. Prescribe topical corticosteroid for ulcers
    D. Administer vitamin K
    E. Provide prophylactic antibiotics
    ANSWER: B
    EXPLANATION: Urgent medical review required; defer dental care until haematologic stability achieved (TG 2025).
  4. Which antibiotic should be avoided in future for this patient?
    A. Metronidazole
    B. Clindamycin
    C. Azithromycin
    D. Doxycycline
    E. Cefalexin
    ANSWER: A
    EXPLANATION: Metronidazole potentiates warfarin → bleeding risk; TG 2025 warns to avoid in patients on anticoagulants.
  5. Ethical responsibility for interprofessional communication?
    A. Manage alone and document later
    B. Communicate findings with GP and rheumatologist, document shared care
    C. Inform only patient verbally
    D. Notify family without consent
    E. Avoid reporting adverse drug events
    ANSWER: B
    EXPLANATION: AHPRA Code mandates collaborative care and documentation when systemic drug interactions cause oral manifestations.

Clinical scenario 7

Questions 31 through 35 refer to the following information

QIMAGE:

A 55-year-old woman presents with swollen gums, lip oedema, and discomfort that developed gradually over three weeks. She reports recently starting an antihypertensive medication.
Medical history: rheumatoid arthritis managed with methotrexate and folic acid; no known allergies. She also reports mild cough and wheezing since starting the new drug.
On examination: generalised gingival enlargement with tenderness, mild facial swelling, and erythematous papules on the chest.

  1. Which medication most likely caused these oral and systemic findings?
    A. Methotrexate
    B. Amlodipine
    C. Enalapril
    D. Folic acid
    E. Paracetamol
    ANSWER: C
    EXPLANATION: ACE inhibitors (like enalapril) can cause angio-oedema and oral mucosal changes. Cough and swelling confirm ACE-related reaction (TG 2025).
  2. Immediate management for this patient?
    A. Continue medication and monitor
    B. Stop enalapril and refer to GP for urgent review
    C. Prescribe chlorhexidine rinse only
    D. Administer steroid mouthwash
    E. Start antibiotics for infection
    ANSWER: B
    EXPLANATION: Drug-induced angio-oedema can compromise the airway; the drug must be ceased and the GP notified immediately (TG 2025).
  3. If amlodipine were the culprit, what mechanism leads to gingival enlargement?
    A. Inflammatory infiltrate in lamina propria
    B. Fibroblast proliferation with excess collagen
    C. Reduced vascularity and necrosis
    D. Epithelial atrophy
    E. Immunologic hypersensitivity
    ANSWER: B
    EXPLANATION: Calcium channel blockers increase fibroblast activity → collagen accumulation and gingival hyperplasia.
  4. What is the safest replacement antihypertensive in this case?
    A. Another ACE inhibitor
    B. Losartan (angiotensin receptor blocker)
    C. Atenolol
    D. Verapamil
    E. Diltiazem
    ANSWER: B
    EXPLANATION: Losartan avoids bradykinin-mediated angio-oedema, providing safe substitution (TG 2025).
  5. Ethical responsibility once the drug reaction is recognised?
    A. Do nothing beyond treatment
    B. Notify the GP and report adverse drug event to TGA
    C. Inform only the patient verbally
    D. Stop medication and manage alone
    E. Note in records but no further action
    ANSWER: B
    EXPLANATION: AHPRA Code mandates notification to prescriber and reporting significant adverse reactions to TGA for patient safety.

Clinical scenario 8

Questions 36 through 40 refer to the following information

QIMAGE:

A 70-year-old man with poorly controlled diabetes presents with right-sided facial pain and dark discoloration of his palate.
Medical history: Type 2 diabetes (HbA1c 8.9%), hypertension, chronic sinusitis treated recently with oral prednisolone, and stage 3 chronic kidney disease.
On examination: necrotic ulcer on hard palate, periorbital swelling, and facial numbness. Temperature 38.6 °C, blood glucose 18 mmol/L.

  1. What is the most likely diagnosis?
    A. Bacterial sinusitis
    B. Mucormycosis
    C. Actinomycosis
    D. Cavernous sinus thrombosis
    E. Aspergillosis
    ANSWER: B
    EXPLANATION: Necrotic palate with facial swelling in an immunocompromised diabetic → mucormycosis (TG 2025).
  2. Immediate management?
    A. Oral fluconazole 50 mg daily
    B. Urgent hospital referral for IV amphotericin B and surgical debridement
    C. Extraction of affected tooth in clinic
    D. Prescribe amoxicillin + metronidazole
    E. Topical antifungal mouthrinse
    ANSWER: B
    EXPLANATION: Mucormycosis is a life-threatening emergency requiring hospital IV antifungal and surgical intervention (TG 2025).
  3. Which predisposing factor is most significant?
    A. Hypertension
    B. Corticosteroid therapy and hyperglycaemia
    C. Sinusitis alone
    D. Dehydration
    E. Chronic kidney disease
    ANSWER: B
    EXPLANATION: Steroid use + uncontrolled diabetes greatly increase risk of invasive fungal infection (TG 2025).
  4. Which antifungal formulation minimises renal toxicity?
    A. Amphotericin B conventional
    B. Liposomal amphotericin B
    C. Nystatin rinse
    D. Ketoconazole tablets
    E. Topical clotrimazole
    ANSWER: B
    EXPLANATION: Liposomal form reduces nephrotoxicity; recommended for diabetic patients with CKD (TG 2025).
  5. What adjunctive measure improves survival?
    A. Increase corticosteroid dose
    B. Tight glycaemic control and prompt surgical debridement
    C. Broad-spectrum antibiotics only
    D. Delay surgery until afebrile
    E. High-dose vitamin C
    ANSWER: B
    EXPLANATION: Early surgical debridement and glucose control are vital for survival (TG 2025).

Clinical scenario 9

Questions 41 through 45 refer to the following information

QIMAGE:

A 74-year-old man presents with sudden swelling on the right side of his face and drooping of the right eyelid. He reports severe pain radiating to his temple and behind the eye for the last 24 hours.
Past history: Type 2 diabetes, ischaemic heart disease (post-stent 3 years ago), and hypertension. Current medications: metformin 1 g BD, atenolol 50 mg OD, ramipril 5 mg OD, aspirin 100 mg OD.
On examination: right-sided periorbital oedema, proptosis, chemosis, and restricted ocular movement. Oral exam reveals periapical infection around tooth 47. Temperature 38.4 °C.

  1. What is the most likely diagnosis?
    A. Orbital cellulitis
    B. Cavernous sinus thrombosis
    C. Maxillary sinusitis
    D. Osteomyelitis of mandible
    E. Facial palsy
    ANSWER: B
    EXPLANATION: Facial infection with ocular signs (proptosis, ophthalmoplegia) indicates cavernous sinus thrombosis, a life-threatening dental complication (TG 2025).
  2. What is the immediate management?
    A. Prescribe oral antibiotics and review
    B. Urgent hospital referral for IV antibiotics and imaging
    C. Drain periapical abscess in clinic
    D. Start high-dose oral steroids
    E. Give analgesics and observe
    ANSWER: B
    EXPLANATION: Cavernous sinus thrombosis requires hospital IV antibiotics (e.g. ceftriaxone + metronidazole) and neuro-ophthalmic evaluation (TG 2025).
  3. Which imaging study confirms the diagnosis?
    A. Orthopantomogram
    B. Contrast-enhanced MRI
    C. Periapical radiograph
    D. CBCT
    E. CT chest
    ANSWER: B
    EXPLANATION: MRI with contrast detects venous thrombosis and soft-tissue spread; dental imaging is inadequate.
  4. Which potential drug interaction should be considered during inpatient therapy?
    A. Metronidazole + Aspirin
    B. Metronidazole + Warfarin (if anticoagulated)
    C. Ceftriaxone + Atenolol
    D. Ramipril + Paracetamol
    E. Amoxicillin + Metformin
    ANSWER: B
    EXPLANATION: Metronidazole potentiates warfarin’s anticoagulant effect; INR must be closely monitored (TG 2025).
  5. Ethical duty of the treating dentist?
    A. Manage conservatively without referral
    B. Immediate hospital referral and documentation of clinical findings
    C. Wait for imaging before referral
    D. Prescribe antibiotics and review in 48 h
    E. Refer only if patient insists
    ANSWER: B
    EXPLANATION: AHPRA 2024 Code mandates urgent referral, full documentation, and communication with medical team when airway or CNS involvement is suspected.

Clinical scenario 10

Questions 46 through 50 refer to the following information

QIMAGE:

A 61-year-old man presents with fatigue, easy bruising, and bleeding gums for two weeks. He recently began taking linezolid 600 mg BD for a resistant skin infection prescribed by his GP.
Past medical history: hypertension on atenolol, Type 2 diabetes on metformin, and chronic kidney disease stage 3. No known drug allergies.
On examination: pale mucosa, petechiae on the palate, gingival bleeding, and mild tachycardia.

  1. What is the most likely cause of his symptoms?
    A. Vitamin C deficiency
    B. Linezolid-induced bone-marrow suppression
    C. Chronic kidney disease anaemia
    D. Thrombocytopenia from aspirin
    E. Leukaemia
    ANSWER: B
    EXPLANATION: Linezolid can cause pancytopenia after > 10 days therapy; TG 2025 lists bone-marrow suppression as serious adverse effect.
  2. Which blood test confirms this diagnosis?
    A. Full blood count showing low platelets and neutrophils
    B. Coagulation profile
    C. Renal function test
    D. HbA1c
    E. Liver enzymes
    ANSWER: A
    EXPLANATION: Cytopenias on FBC confirm myelosuppression; coagulation profile may be normal.
  3. What is the appropriate dental management?
    A. Proceed with extractions cautiously
    B. Defer all invasive care and refer urgently to physician
    C. Prescribe chlorhexidine rinse only
    D. Give vitamin K injection
    E. Start antibiotics prophylactically
    ANSWER: B
    EXPLANATION: Invasive dental care contraindicated until marrow recovers; urgent medical review essential (TG 2025).
  4. If systemic infection develops, which antibiotic should be avoided due to additive toxicity?
    A. Clindamycin
    B. Amoxicillin
    C. Gentamicin
    D. Doxycycline
    E. Cefalexin
    ANSWER: C
    EXPLANATION: Gentamicin + linezolid increases nephro- and neuro-toxicity; avoid in CKD stage 3 (TG 2025).
  5. Ethical responsibility for ongoing care documentation?
    A. Continue without informing GP
    B. Communicate findings with prescriber and document adverse reaction
    C. Record only treatment outcome
    D. Report directly to TGA without patient consent
    E. Avoid written record
    ANSWER: B
    EXPLANATION: AHPRA 2024 requires inter-professional communication and formal documentation of drug adverse events to ensure continuity of care.

Clinical scenario 11

Questions 51 through 55 refer to the following information

QIMAGE:

A 79-year-old woman presents with multiple painful mouth ulcers, facial rash, and malaise. Two weeks ago, she was prescribed trimethoprim–sulfamethoxazole for a urinary tract infection.
Medical history: atrial fibrillation on warfarin, hypertension on perindopril, and osteoarthritis managed with paracetamol.
On examination: haemorrhagic crusting of the lips, target-like lesions on trunk, and erosive ulcerations of buccal mucosa. Temperature 38.2 °C.

  1. What is the most likely diagnosis?
    A. Herpetic stomatitis
    B. Erythema multiforme major (Stevens–Johnson syndrome)
    C. Drug-induced lichenoid reaction
    D. Aphthous ulcers
    E. Pemphigus vulgaris
    ANSWER: B
    EXPLANATION: Widespread mucocutaneous ulceration and target lesions after sulfonamide use indicate SJS (TG 2025).
  2. What is the immediate dental management?
    A. Provide topical corticosteroid mouthwash and review
    B. Cease suspected drug and refer urgently to hospital
    C. Prescribe antibiotics and analgesics
    D. Debride necrotic areas under LA
    E. Observe for a few days
    ANSWER: B
    EXPLANATION: Stevens–Johnson syndrome is a medical emergency requiring hospital admission, fluid management, and systemic therapy (TG 2025).
  3. Which histopathological finding confirms the diagnosis?
    A. Intraepithelial clefting with acantholysis
    B. Subepidermal bullae with necrotic epithelium
    C. Lymphocytic band-like infiltrate
    D. Hyperkeratosis and epithelial thickening
    E. Granulomatous inflammation
    ANSWER: B
    EXPLANATION: Subepidermal blister formation and necrosis confirm SJS/TEN; distinguishes from pemphigus (TG 2025).
  4. What oral care measure supports healing?
    A. Alcohol-based mouthrinse BID
    B. 0.2 % chlorhexidine rinse with topical anaesthetic gel
    C. Hydrogen peroxide mouthwash
    D. Dry brushing to remove crusts
    E. Topical steroid alone
    ANSWER: B
    EXPLANATION: Gentle antiseptic rinses and analgesic gels minimise infection and pain; alcohol and oxidising agents aggravate lesions (TG 2025).
  5. Ethical responsibility after identifying this adverse drug reaction?
    A. Record but take no further action
    B. Notify the GP and report to the TGA Adverse Event System
    C. Treat locally without communication
    D. Prescribe alternative antibiotics independently
    E. Ignore as resolved
    ANSWER: B
    EXPLANATION: AHPRA 2024 and TGA protocols require notification and inter-professional communication after severe ADRs.

Clinical scenario 12

Questions 56 through 60 refer to the following information

QIMAGE:

A 59-year-old man undergoing restorative treatment suddenly becomes pale, clammy, and complains of severe chest pain radiating to the left arm.
Past medical history: previous myocardial infarction (2 years ago), hypertension, diabetes, and hyperlipidaemia.
Current medications: aspirin 100 mg OD, metoprolol 50 mg BD, ramipril 5 mg OD, atorvastatin 20 mg OD, and GTN spray PRN.
The episode began during administration of local anaesthetic containing adrenaline.

  1. What is the most likely diagnosis?
    A. Anxiety attack
    B. Acute coronary syndrome
    C. Vasovagal syncope
    D. Adrenaline overdose
    E. Hypoglycaemia
    ANSWER: B
    EXPLANATION: Crushing chest pain radiating to arm with diaphoresis → ACS; TG 2025 identifies this as a dental emergency.
  2. What is the correct immediate management (ADA 2025 emergency protocol)?
    A. Stop procedure, give GTN spray, oxygen, aspirin 300 mg, and call 000
    B. Continue procedure and provide paracetamol
    C. Lay supine and give ammonia inhalant
    D. Inject adrenaline IM immediately
    E. Give ibuprofen and observe
    ANSWER: A
    EXPLANATION: Standard MI protocol: stop procedure, GTN 400 µg sublingual, oxygen 15 L/min, aspirin 300 mg, call emergency (ADA 2025).
  3. Which pharmacological interaction contributed to the episode?
    A. Adrenaline + β-blocker (metoprolol) causing hypertension and reflex bradycardia
    B. Ramipril + Adrenaline causing hypotension
    C. Aspirin + Adrenaline causing bleeding
    D. Atorvastatin + Lidocaine causing arrhythmia
    E. Metformin + Adrenaline causing hypoglycaemia
    ANSWER: A
    EXPLANATION: Non-selective β-blockers antagonise β₂-vasodilation, leaving α-adrenergic effects → hypertension, reflex bradycardia (TG 2025).
  4. What modification should be applied for future dental care?
    A. Avoid all LAs
    B. Use plain mepivacaine LA, short morning appointments, stress control
    C. Continue adrenaline-containing LA without restriction
    D. Delay all treatment indefinitely
    E. Use GA routinely
    ANSWER: B
    EXPLANATION: TG 2025 recommends stress-free morning appointments with minimal adrenaline (≤ 2 cartridges) in cardiac patients.
  5. Ethical responsibility post-event?
    A. Continue care after symptoms subside
    B. Cease treatment, document event, and send detailed referral to physician
    C. Inform only family verbally
    D. Omit record to avoid liability
    E. Discharge patient without report
    ANSWER: B
    EXPLANATION: AHPRA 2024 Code mandates event documentation, emergency referral, and communication with the treating physician.

Clinical scenario 13

Questions 61 through 65 refer to the following information

QIMAGE:

A 53-year-old male with insulin-dependent diabetes presents for a 90-minute restorative appointment. Midway through the procedure, he becomes pale, anxious, and tremulous. His assistant reports that he has not eaten breakfast.
Medical history: Type 1 diabetes on glargine (long-acting) and aspart (rapid-acting) insulin, hypothyroidism on thyroxine, and mild hypertension.
On examination: sweating, confusion, pulse 110/min, BP 122/76 mm Hg.

  1. What is the most likely diagnosis?
    A. Adrenaline overdose
    B. Hypoglycaemia
    C. Vasovagal syncope
    D. Hyperglycaemia
    E. Panic attack
    ANSWER: B
    EXPLANATION: Missed meal + insulin + tremor and confusion → classic hypoglycaemia (TG 2025).
  2. What is the correct immediate management?
    A. Give 15 g oral glucose if conscious
    B. Call ambulance first
    C. Lay supine with legs raised
    D. Give IM adrenaline 0.5 mg
    E. Administer insulin correction
    ANSWER: A
    EXPLANATION: Conscious hypoglycaemia → fast-acting carbohydrate; if unconscious, administer glucagon IM (TG 2025).
  3. What mechanism explains tremor and tachycardia in this episode?
    A. Cortisol excess
    B. Adrenergic response to low glucose
    C. Hypovolaemia
    D. Acetylcholine release
    E. Elevated thyroid hormone
    ANSWER: B
    EXPLANATION: Catecholamine release (adrenaline) produces autonomic symptoms during hypoglycaemia.
  4. When should future dental appointments be scheduled?
    A. Early morning before insulin dose
    B. After normal meal and insulin
    C. Late afternoon after fasting
    D. Late evening to monitor
    E. Immediately post-exercise
    ANSWER: B
    EXPLANATION: TG 2025 recommends mid-morning appointments after normal meal and insulin to minimise hypoglycaemia risk.
  5. Ethical and professional responsibility for incident management?
    A. Document, inform GP, and educate patient on meal timing
    B. Provide glucose and discharge without notes
    C. Continue treatment immediately
    D. Keep event confidential and unrecorded
    E. Refer only if hospitalised
    ANSWER: A
    EXPLANATION: AHPRA Code mandates detailed documentation and communication with GP to prevent recurrence.

Clinical scenario 14

Questions 66 through 70 refer to the following information

QIMAGE:

A 76-year-old woman presents for extraction of tooth 26 due to root fracture. She reports mild shortness of breath when lying flat.
Medical history: chronic heart failure (NYHA Class II), hypertension, and osteoarthritis.
Medications: furosemide 40 mg OD, digoxin 125 µg OD, ramipril 5 mg OD, and paracetamol PRN.
On examination: pulse 58 bpm irregular, BP 108/70 mm Hg, mild ankle oedema, and jugular venous distension.

  1. What is the most likely cause of her slow pulse?
    A. Furosemide effect
    B. Digoxin toxicity
    C. Ramipril adverse reaction
    D. Beta-blocker use
    E. Hypothyroidism
    ANSWER: B
    EXPLANATION: Loop diuretics (furosemide) cause hypokalaemia → increases digoxin toxicity presenting with bradycardia and arrhythmia (TG 2025).
  2. Which symptom most strongly indicates digoxin toxicity?
    A. Nausea and vomiting
    B. Blurred vision and yellow-green halos
    C. Fatigue
    D. Diarrhoea
    E. Myalgia
    ANSWER: B
    EXPLANATION: Xanthopsia (yellow-green visual halos) is classic for digoxin toxicity.
  3. If dental infection develops, which antibiotic should be avoided?
    A. Erythromycin
    B. Amoxicillin
    C. Clindamycin
    D. Doxycycline
    E. Cefalexin
    ANSWER: A
    EXPLANATION: Macrolides inhibit P-glycoprotein → increase digoxin serum concentration; avoid erythromycin (TG 2025).
  4. What is the optimal dental chair position for this patient?
    A. Fully supine
    B. Semi-supine at 45°
    C. Upright at 90°
    D. Trendelenburg position
    E. Prone position
    ANSWER: B
    EXPLANATION: Semi-supine position minimises pulmonary congestion and dyspnoea (TG 2025).
  5. Ethical duty before proceeding with dental treatment?
    A. Proceed without consultation
    B. Confirm medical stability and communicate with cardiologist
    C. Delay extraction indefinitely
    D. Stop all cardiac medications
    E. Obtain verbal consent only
    ANSWER: B
    EXPLANATION: AHPRA 2024 Code mandates inter-professional collaboration and written documentation before treating medically compromised patients.